Dermatology Flashcards

1
Q

What is the management of extensive infection by dermatophytes?

A
  • Oral terbinafine
How well did you know this?
1
Not at all
2
3
4
5
Perfectly
2
Q

How should a child with new-onset purpura be managed?

A

Immediate referral to secondary care to exclude ALL

How well did you know this?
1
Not at all
2
3
4
5
Perfectly
3
Q

Where are keloid scars most common?

A

Sternum

How well did you know this?
1
Not at all
2
3
4
5
Perfectly
4
Q

What is the management of athletes foot?

A
  • Topical miconazole
  • terbinafine
How well did you know this?
1
Not at all
2
3
4
5
Perfectly
5
Q

What medications exacerbate plaque psoriasis?

A

Beta blockers, lithium, antimalarias, NSAIDS and ACE

How well did you know this?
1
Not at all
2
3
4
5
Perfectly
6
Q

What should be used for long-term psoriasis?

A

Calcipotriol

How well did you know this?
1
Not at all
2
3
4
5
Perfectly
7
Q

What is pemphigoid gestationis?

A

Pruritic blistering lesions which in the peri-umbilical region and can then spread to trunk, back buttocks and arms in the 2nd/3rd trimester

How well did you know this?
1
Not at all
2
3
4
5
Perfectly
8
Q

When does pemphigoid gestationis present?

A

In the second/third pregnancy

How well did you know this?
1
Not at all
2
3
4
5
Perfectly
9
Q

How is pemphigoid gestationis managed?

A

Oral corticosteroids

How well did you know this?
1
Not at all
2
3
4
5
Perfectly
10
Q

What can make perioral dermatitis worse?

A

Topical steroids

How well did you know this?
1
Not at all
2
3
4
5
Perfectly
11
Q

What is the first line management of venous ulcers?

A

Compression bandaging

How well did you know this?
1
Not at all
2
3
4
5
Perfectly
12
Q

What is a rare side effects of penicllins?

A

Toxic epidermal necrolysis

How well did you know this?
1
Not at all
2
3
4
5
Perfectly
13
Q

What is the most common malignancy associated with acanthosis nigricans?

A

gastrointestinal adenocarcinoma

How well did you know this?
1
Not at all
2
3
4
5
Perfectly
14
Q

What are some causes of acanthosis nigricans?

A
  • T2DM
  • GI cancer
  • PCOS
  • Obesity
  • Cushing’s disease
  • Acromegaly
  • COCP, nicotinic acid
  • Hypothyroidism
  • Familial
How well did you know this?
1
Not at all
2
3
4
5
Perfectly
15
Q

Acanthosis nigricans?

A

symmetrical, brown, velvety plaques that often found on neck, axilla and groin

How well did you know this?
1
Not at all
2
3
4
5
Perfectly
16
Q
A
How well did you know this?
1
Not at all
2
3
4
5
Perfectly
17
Q

Pathophysiology of acanthosis nigricans?

A

insulin resistance → hyperinsulinemia → stimulation of keratinocytes and dermal fibroblast proliferation via interaction with insulin-like growth factor receptor-1 (IGFR1)

How well did you know this?
1
Not at all
2
3
4
5
Perfectly
18
Q

purple, polygonal, pruritic papules suggest what?

A

Lichen planus

How well did you know this?
1
Not at all
2
3
4
5
Perfectly
19
Q

What is a port wine stain?

A

A vascular birthmark: deep red or purple colour usually over the cheek

How well did you know this?
1
Not at all
2
3
4
5
Perfectly
20
Q

What is a strawberry naevus?

A

A soft raised vascular swelling which is bright red in colour

How well did you know this?
1
Not at all
2
3
4
5
Perfectly
21
Q

What is a salmon patch?

A

A flat dull-red area usually on the face and neck, usually in the midline which causes no symptoms

How well did you know this?
1
Not at all
2
3
4
5
Perfectly
22
Q

What derm feature is associated with Sturge Webber syndrome?

A

Port wine stain

How well did you know this?
1
Not at all
2
3
4
5
Perfectly
23
Q

What is pityriasis versicolor?

A

A superficial fungal (Malassezia) infection which affects the trunk causing pink/brown patches, sometimes after a suntan

How well did you know this?
1
Not at all
2
3
4
5
Perfectly
24
Q

What is the management of pityriasis versicolor?

A

Topical antifungal: ketoconazole

How well did you know this?
1
Not at all
2
3
4
5
Perfectly
25
Q

Salmon patch vs port wine stain?

A

Salmon patch will usually self resolve

How well did you know this?
1
Not at all
2
3
4
5
Perfectly
26
Q

What is toxic epidermal necrolysis?

A

A life-threatening emergency scaled skin appearance usually secondary to a drug reaction

How well did you know this?
1
Not at all
2
3
4
5
Perfectly
27
Q

How does toxic epidermal necrolysis present?

A
  • Systemically unwell patient
  • Positive Nikolsky sign: epidermis separates with mild lateral pressure
How well did you know this?
1
Not at all
2
3
4
5
Perfectly
28
Q

What drugs are known to induce toxic epidermal necrolysis?

A
  • Phenytoin
  • Allopurinol
  • Penicillins
  • NSAIDs
  • Carbamazepine
How well did you know this?
1
Not at all
2
3
4
5
Perfectly
29
Q

How is toxic epidermal necrolysis managed?

A
  • Stop trigger
  • Supportive care: fluid loss and electrolyte derangements are complications
  • IV immunoglobulins are first line
How well did you know this?
1
Not at all
2
3
4
5
Perfectly
30
Q

What is pityriasis rosea?

A

An acute self limiting rash which affects younger people

How well did you know this?
1
Not at all
2
3
4
5
Perfectly
31
Q

How does pityriasis rosea present?

A
  • Herald patch on trunk
  • Followed by smaller red macules
  • Lethargy
  • Christmas tree distribution as on upper parts of arms and legs
How well did you know this?
1
Not at all
2
3
4
5
Perfectly
32
Q

Bowens vs SCC?

A

SCC will ulcerate, grow over weeks-months and may bleed

How well did you know this?
1
Not at all
2
3
4
5
Perfectly
33
Q
A
How well did you know this?
1
Not at all
2
3
4
5
Perfectly
34
Q

Bowen’s disease?

A

precancerous dermatosis that is precursor to SCC (5-10% chance); elderly; red scaly patches 10-15mm in size, slow growing and often on sun exposed areas eg. head, neck, lower limbs

How well did you know this?
1
Not at all
2
3
4
5
Perfectly
35
Q

What is the management of Bowens?

A

Topical 5-fluorouracil 2x day 4w
- can cause inflam and eryth so topical steroids to control

How well did you know this?
1
Not at all
2
3
4
5
Perfectly
36
Q

A rapidly progressing painful rash in children with atopic eczema suggests what?

A

Eczema herpeticum

How well did you know this?
1
Not at all
2
3
4
5
Perfectly
37
Q

How is eczema herpeticum managed?

A

Admission for IV aciclovir with urgent derm and opthal review

How well did you know this?
1
Not at all
2
3
4
5
Perfectly
38
Q

On examination, what does eczema herpeticum look like?

A

monomorphic punched-out erosions (circular, depressed, ulcerated lesions) usually 1-3mm in diameter

How well did you know this?
1
Not at all
2
3
4
5
Perfectly
39
Q

What are causes of erythema nodosum?

A

NO cause
Drugs
OCP
Sarcoidosis
UC and Crohns
Microorganisms such as TB, Strep and Chlamydia

How well did you know this?
1
Not at all
2
3
4
5
Perfectly
40
Q

What is erythema ab igne?

A

A skin disorder associated with over exposure to infrared radiation: hot water bottles or open fires.

Reticulated erythematous patches with hyperpigmentation and telangiectasia.

Can develop SCC if not treated.

How well did you know this?
1
Not at all
2
3
4
5
Perfectly
41
Q

How should patients with moderate/severe papules in rosacea be managed?

A

Topical ivermectin and oral doxycycline

How well did you know this?
1
Not at all
2
3
4
5
Perfectly
42
Q

What is the management of refractory pain in shingles?

A

Prednisolone

How well did you know this?
1
Not at all
2
3
4
5
Perfectly
43
Q

What can iron deficiency anaemia cause?

A

Pruritus

How well did you know this?
1
Not at all
2
3
4
5
Perfectly
44
Q

What is the management of athletes foot?

A
  • Topical imidazole
  • Terbinafine
How well did you know this?
1
Not at all
2
3
4
5
Perfectly
45
Q

What is a complication of burns?

A

Curlings ulcers - acute gastric ulcers which develop in response to physiological stress: usually present with vomiting blood

How well did you know this?
1
Not at all
2
3
4
5
Perfectly
46
Q

What pathogen causes eczema herpeticum?

A

Herpes simplex 1 or 2 causes severe primary infection of the skin

How well did you know this?
1
Not at all
2
3
4
5
Perfectly
47
Q

When to use skin patch vs skin prick test?

A

Skin patch - used for contact dermatitis
Skin prick - used for immediate hypersensitivity reactions e.g. food and resp allergies

How well did you know this?
1
Not at all
2
3
4
5
Perfectly
48
Q

Skin infection occurring 2-4 weeks after a strep infection?

A

Guttate psoriasis

How well did you know this?
1
Not at all
2
3
4
5
Perfectly
49
Q

‘tear drop papule on the trunk and limb’

A

Guttate psoriasis

How well did you know this?
1
Not at all
2
3
4
5
Perfectly
50
Q

What is livedo reticularis?

A
  • A non-blanching, reticulated rash caused by obstruction of the capillaries
  • Can be idiopathic or part of SLE, Ehlers-Danlos
How well did you know this?
1
Not at all
2
3
4
5
Perfectly
51
Q

non-healing painless ulcer associated with a chronic scar

A

Squamous cell carcinoma

How well did you know this?
1
Not at all
2
3
4
5
Perfectly
52
Q

How long will people with shingles be infectious?

A

Until the vesicles have crusted over - 5-7 days after onset

How well did you know this?
1
Not at all
2
3
4
5
Perfectly
53
Q

Steroid treatment in psoriasis?

A

Aim for 4 week break in between courses

How well did you know this?
1
Not at all
2
3
4
5
Perfectly
54
Q

What is the main reason for using antivirals for shingles?

A

Reduce post herpetic neuralgia

How well did you know this?
1
Not at all
2
3
4
5
Perfectly
55
Q

What phenomenon does psoriasis exhibit?

A

Koebner phenomenon - new skin lesions form at the site of cutaneous injury

How well did you know this?
1
Not at all
2
3
4
5
Perfectly
56
Q

What is the most important prognostic factor with malignant melanoma?

A

Breslow’s thickness of the tumour and lymph node status

How well did you know this?
1
Not at all
2
3
4
5
Perfectly
57
Q

intensely pruritic rash on the palms and soles

A

pompholyx eczema

How well did you know this?
1
Not at all
2
3
4
5
Perfectly
58
Q

What term condition can isoniazid cause?

A

Pellagra - dermatitis, diarrhoea and dementia

How well did you know this?
1
Not at all
2
3
4
5
Perfectly
59
Q

How is rosacea managed?

A

Predominant flushing: topical brimonidine
Mild-moderate papules: topical ivermectin
Moderate-severe papules: tropical ivermictin + oral doxycycline

How well did you know this?
1
Not at all
2
3
4
5
Perfectly
60
Q

single well-demarcated, erythematous circular patch with a raised edge and central hypopigmentation

A

tinea corpis

How well did you know this?
1
Not at all
2
3
4
5
Perfectly
61
Q

Management of severe urticaria?

A

Short course of oral steroids plus anti-histamine

How well did you know this?
1
Not at all
2
3
4
5
Perfectly
62
Q

What should be screened for before starting isotretinoin?

A

Mental health conditions/Pregnany

How well did you know this?
1
Not at all
2
3
4
5
Perfectly
63
Q

What is a wheal?

A

Transient, raised lesions due to underlying dermal oedema -> urticaria

How well did you know this?
1
Not at all
2
3
4
5
Perfectly
64
Q

What does histology of eczema show?

A

IgE mediated response
- Epidermal acanthosis
- Hyperkeratosis
- Parakeratosis

How well did you know this?
1
Not at all
2
3
4
5
Perfectly
65
Q

red scaling plaques in sun exposed areas of skin

A

Discoid lupus

How well did you know this?
1
Not at all
2
3
4
5
Perfectly
66
Q

Actinic keratosis vs sebhorreic keratosis?

A

AK - lesions lighter in colour + more in line with the skin, SK - lesions will be much darker + stuck on appearance

How well did you know this?
1
Not at all
2
3
4
5
Perfectly
67
Q

What can be used for patients with severe childhood eczema?

A

Wet wrapping

How well did you know this?
1
Not at all
2
3
4
5
Perfectly
68
Q

Flu-illness, dry cough, target shaped lesions + anaemia

A

Erythema multiforme caused by Mycoplasma pneumonia

How well did you know this?
1
Not at all
2
3
4
5
Perfectly
69
Q

Which organisms cause necrotising fasciitis?

A

Type 1 - mixed
Type 2 - strep pyogenes

How well did you know this?
1
Not at all
2
3
4
5
Perfectly
70
Q

What are risk factors for malignant melanoma?

A
  • Fair complexion: pale skin (Fitzpatrick Skin Type I and II), light hair, light eyes
  • FH
  • Sunburn
  • Sun exposure
  • outdoor occupation
  • immunosupression
How well did you know this?
1
Not at all
2
3
4
5
Perfectly
71
Q

What are sites where malignant melanoma can occur?

A
  • Choroid of the eye
  • CNS
  • GI tract
  • Neck
How well did you know this?
1
Not at all
2
3
4
5
Perfectly
72
Q

What are features of BCC?

A
  • Small, pearly white lesion
  • Rolled edge
  • Central ulcer
  • Can be pigmented
How well did you know this?
1
Not at all
2
3
4
5
Perfectly
73
Q

What surgical technique can be used for BCC?

A

Mohs micrographic surgery

How well did you know this?
1
Not at all
2
3
4
5
Perfectly
74
Q

What is the pathophysiology of acne?

A

Increased sebum production leads to the pilosebaceous follicles becoming blocked and infected

How well did you know this?
1
Not at all
2
3
4
5
Perfectly
75
Q

Which bacteria is involved with acne?

A

Propionbacterium acnes

How well did you know this?
1
Not at all
2
3
4
5
Perfectly
76
Q

What are S/E of isotretinoin?

A

Dry skin, dry eyes, depression, migraine, muscle aches

How well did you know this?
1
Not at all
2
3
4
5
Perfectly
77
Q

What is the name for SCC carcinoma in situ?

A

Bowens

How well did you know this?
1
Not at all
2
3
4
5
Perfectly
78
Q

What anatomical sites on head and neck give worst prognosis for SCC?

A

Ear and lip

How well did you know this?
1
Not at all
2
3
4
5
Perfectly
79
Q

What is the management of lichen sclerosus?

A

Topical steroids and emollients

How well did you know this?
1
Not at all
2
3
4
5
Perfectly
80
Q

What does lichen sclerosus increase the risk of

A

Vulval cancer

How well did you know this?
1
Not at all
2
3
4
5
Perfectly
81
Q

Most common organism for cellulitis?

A

Strep pyogenes then staph aureus

How well did you know this?
1
Not at all
2
3
4
5
Perfectly
82
Q

Incredibly itchy blisters + papules in someone with coeliac?

A

Dermatitis herpetiformis

How well did you know this?
1
Not at all
2
3
4
5
Perfectly
83
Q

What classification system is used for patients with cellulitis?

A

Eron

How well did you know this?
1
Not at all
2
3
4
5
Perfectly
84
Q

What is the Abx of choice for severe cellulitis?

A

Co-Amox / Clindamycin

How well did you know this?
1
Not at all
2
3
4
5
Perfectly
85
Q

Target lesions on back of hands/feet spreading to limbs?

A

Erythema multiforme

How well did you know this?
1
Not at all
2
3
4
5
Perfectly
86
Q

What is erythema nodosum?

A

Painful inflammation of the sub cut fat more common in females

How well did you know this?
1
Not at all
2
3
4
5
Perfectly
87
Q

Lichen planus vs lichen sclerosus

A

Planus - can affect inside the vagina, Sclerosus will only affect the external genitals
Planus - purpuric papular lesions, Sclerosus - small, white plaques

How well did you know this?
1
Not at all
2
3
4
5
Perfectly
88
Q

‘saw tooth pattern of epidermal hyperplasia, T-cell infiltration and reduced melanocytes’

A

Lichen planus

How well did you know this?
1
Not at all
2
3
4
5
Perfectly
89
Q

Round, pearly papules in crops in children?

A

Think Molluscum - no treatment needed

How well did you know this?
1
Not at all
2
3
4
5
Perfectly
90
Q

Erythema multiforme vs pityriasis vericolor?

A

Pityriasis - white/brown patches with dry scaly skin usually on trunk
Erythema - red, round lesions

How well did you know this?
1
Not at all
2
3
4
5
Perfectly
91
Q

Papular rash around abdomen/wrist/inner thigh which itches worse at night?

A

Scabies - treat with Permethrin for all household members

How well did you know this?
1
Not at all
2
3
4
5
Perfectly
92
Q

Flaky rash on face/scalp/trunk which is not itchy?

A

Seborrheic Dermatitis

How well did you know this?
1
Not at all
2
3
4
5
Perfectly
93
Q

Which fungus causes seborrheic dermatitis?

A

Malassezia furfur

How well did you know this?
1
Not at all
2
3
4
5
Perfectly
94
Q

What conditions are associated with seborrheic dermatitis?

A

Parkinsons / HIV

How well did you know this?
1
Not at all
2
3
4
5
Perfectly
95
Q

What are common complications of seborrheic dermatitis?

A

Otitis externa / Blepharitis

How well did you know this?
1
Not at all
2
3
4
5
Perfectly
96
Q

Most common and most aggressive form of melanoma?

A

Common - superficial spreading
Aggressive - Nodular

How well did you know this?
1
Not at all
2
3
4
5
Perfectly
97
Q

What is the management of actinic keratoses?

A
  • Avoid sun
  • Fluorouracil cream
How well did you know this?
1
Not at all
2
3
4
5
Perfectly
98
Q

What is erythroderma?

A

Complex process leading to rapid epidermal cell turnover -> large desquamated area which is red and painful

How well did you know this?
1
Not at all
2
3
4
5
Perfectly
99
Q

What is the management of erythroderma?

A

Urgent derm referral with admission to burns unit for emollients, fluids and wet balances

How well did you know this?
1
Not at all
2
3
4
5
Perfectly
100
Q

What can often cause erythroderma?

A

Drugs like sulphonylureas, isoniazid and sulphonamides

How well did you know this?
1
Not at all
2
3
4
5
Perfectly
101
Q

Which antibiotic causes red man syndrome?

A

Vancomycin

How well did you know this?
1
Not at all
2
3
4
5
Perfectly
102
Q

Signs of psoriasis on nails?

A
  • Nail pitting
  • Oncholysis
  • Oil drop sign
How well did you know this?
1
Not at all
2
3
4
5
Perfectly
103
Q

What sign in psoriasis where pinpoint bleeding occurs when scales are scraped?

A

Auspitz sign

How well did you know this?
1
Not at all
2
3
4
5
Perfectly
104
Q

What are the ABCDE of lesions?

A

Asymmetry
Border
Colour change
Diameter > 6mm
Evolving lesion

How well did you know this?
1
Not at all
2
3
4
5
Perfectly
105
Q

What are cardinal signs of inflammation?

A

Dolor
Calor
Rubor
Tumour
Loss of function

How well did you know this?
1
Not at all
2
3
4
5
Perfectly
106
Q

What autoimmune diseases are associated with dermatitis herpetiformis?

A
  • Coeliac
  • Vitiligo
  • Addisons
  • T1DM
How well did you know this?
1
Not at all
2
3
4
5
Perfectly
107
Q

What are skin manifestations of liver disease?

A
  • Flushing
  • Hyperpigmentation
  • Palmar erythema
  • Jaundice
  • Spider naevi
How well did you know this?
1
Not at all
2
3
4
5
Perfectly
108
Q

What are nail manifestations of liver disease?

A
  • Clubbing
  • Koilonychia
  • Leukonychia
How well did you know this?
1
Not at all
2
3
4
5
Perfectly
109
Q

What is seborrheic keratosis?

A

Benign warty epidermal growths which occur in older patients - can be removed by cryotherapy/curettage

How well did you know this?
1
Not at all
2
3
4
5
Perfectly
110
Q

What is folliculitis?

A

Inflammatory condition of hair follicles caused by Staph aureus

How well did you know this?
1
Not at all
2
3
4
5
Perfectly
111
Q

Crusty lesion which may bleed easily when bumped or scratched?

A

SCC

How well did you know this?
1
Not at all
2
3
4
5
Perfectly
112
Q

What type of hypersensitivity reaction is scabies?

A

delayed type IV

How well did you know this?
1
Not at all
2
3
4
5
Perfectly
113
Q

How long can pruritus persist for with scabies?

A

6 weeks

How well did you know this?
1
Not at all
2
3
4
5
Perfectly
114
Q

Red or black lump, oozes or bleeds, sun-exposed skin

A

Nodular melanoma

How well did you know this?
1
Not at all
2
3
4
5
Perfectly
115
Q

Acute onset of tear-drop scaly papules on trunk and limbs

A

Guttate psoriasis

How well did you know this?
1
Not at all
2
3
4
5
Perfectly
116
Q

Afro-Carribean with nodule not on sun exposed area?

A

Think acral lentiginous melanoma

How well did you know this?
1
Not at all
2
3
4
5
Perfectly
117
Q

How often should people with scabies be treated?

A

Twice with treatments one week apart

How well did you know this?
1
Not at all
2
3
4
5
Perfectly
118
Q

PUVA therapy is associated with what?

A

SCC

How well did you know this?
1
Not at all
2
3
4
5
Perfectly
119
Q

Hard swelling near the umbilicus?

A

Think Sister Mary Joseph nodule - gastric cancer

How well did you know this?
1
Not at all
2
3
4
5
Perfectly
120
Q

Rash improving with sunlight?

A

Psoriasis

How well did you know this?
1
Not at all
2
3
4
5
Perfectly
121
Q

Small lower limb ulcer which develops into deep ulcer following minor trauma?

A

Think pyoderma gangrenosum - treat with prednisolone

How well did you know this?
1
Not at all
2
3
4
5
Perfectly
122
Q

What is pyoderma gangrenosum associated with?

A
  • Diabetes
  • IBD
  • GPA
  • Sarcoidosis
  • Thyroid disorders
How well did you know this?
1
Not at all
2
3
4
5
Perfectly
123
Q

widespread raised pink/pearly white papules with a central umbilication on his trunk, face, hands, legs and feet

A

Molluscum contagiosum

caused by Molluscum contagagiosum virus

Reassure: spontaneous resolution within 18m; but contagious so avoid sharing towels, clothes ect. DONT exclude from school.

If HIV +ve with extensive lesions then urgent referral to HIV specialist; also refer if on/around eyes or genitalia (?STI)

If lesions itchy or troublesome then either squeeze and bath or cryotherapy if older.

How well did you know this?
1
Not at all
2
3
4
5
Perfectly
124
Q

What can persistent molluscum be a sign of?

A

Immunocompromise -> HIV testing needed

How well did you know this?
1
Not at all
2
3
4
5
Perfectly
125
Q

Layers of the skin?

A
  • Epidermis
  • Dermis
  • Subcut tissue
How well did you know this?
1
Not at all
2
3
4
5
Perfectly
126
Q

Topical corticosteroids can cause what?

A

Skin depigmentation in patients with darker skin

How well did you know this?
1
Not at all
2
3
4
5
Perfectly
127
Q

Bullous pemphigoid vs pemphigus vulgaris?

A

BP - elderly patient with deep blisters and no mucosal involvement -> treat with topical steroids
PV - middle aged patient with superficial blisters, mucousal involvement -> treat with oral steroids/immunosuppression

How well did you know this?
1
Not at all
2
3
4
5
Perfectly
128
Q

pruritic wheals?

A

Urticaria

How well did you know this?
1
Not at all
2
3
4
5
Perfectly
129
Q

red, scaly rash on the face and scalp which is itchy

A

Sebhorreic dermatitis

How well did you know this?
1
Not at all
2
3
4
5
Perfectly
130
Q

What is the most common infection which cause erythema multiforme?

A

Herpes simplex virus

How well did you know this?
1
Not at all
2
3
4
5
Perfectly
131
Q

What is vitiligo?

A

Autoimmune condition resulting in loss of melanocytes and consequent depigmentation of skin; develops by 20-30yrs; associated with DMT1, Addison’s, autoimmune thyroid disorders, pernicious anaemia and alopecia areata.

How well did you know this?
1
Not at all
2
3
4
5
Perfectly
132
Q

What causes erythroderma?

A
  • Dermatitis
  • Psoriasis
  • Drug allergies
  • Idiopathic
How well did you know this?
1
Not at all
2
3
4
5
Perfectly
133
Q

What is the management of keratoacanthomas?

A

Surgical excision

How well did you know this?
1
Not at all
2
3
4
5
Perfectly
134
Q

purpura, abdominal pain, and arthritis

A

Henoch-Schonlein Purpura - IgA mediated vasculitis triggered by infection

How well did you know this?
1
Not at all
2
3
4
5
Perfectly
135
Q

coin-shaped lesions that are well-demarcated, with exudates and crusting

A

Discoid eczema

How well did you know this?
1
Not at all
2
3
4
5
Perfectly
136
Q

Steroid makes a rash worse?

A

Think fungal causes

How well did you know this?
1
Not at all
2
3
4
5
Perfectly
137
Q

Deep pustular ulcer on leg which is very painful?

A

Think pyoderma gangrenosum

How well did you know this?
1
Not at all
2
3
4
5
Perfectly
138
Q

When to do wide local excision vs Mohs micrographic surgery?

A

Mohs used for lesions where tissue loss should be minimised e.g. face

How well did you know this?
1
Not at all
2
3
4
5
Perfectly
139
Q

Bullous pemphigoid has antibodies against what?

A

Epithelial basement membrane; against hemidesmosomal proteins BP180 and BP230

How well did you know this?
1
Not at all
2
3
4
5
Perfectly
140
Q

Bullous pemphigoid?

A

autoimmune condition causing sub-epidermal blistering of skin.

elderly

itchy tense blisters around flexures typically; usually heal without scarring; NO mucosal invl.

How well did you know this?
1
Not at all
2
3
4
5
Perfectly
141
Q

Actinic keratosis is a precursor for what?

A

Squamous cell carcinoma

How well did you know this?
1
Not at all
2
3
4
5
Perfectly
141
Q

Sebhorreic keratosis is associated with what?

A

Gastric/Colorectal adenocarcinoma

How well did you know this?
1
Not at all
2
3
4
5
Perfectly
141
Q
A
How well did you know this?
1
Not at all
2
3
4
5
Perfectly
142
Q

Most common cause of erythroderma?

A

Skin condition such as psoriasis or eczema

How well did you know this?
1
Not at all
2
3
4
5
Perfectly
143
Q

Staph scalded skin vs TEN

A

TEN will have mucosal involvement - usually oral

How well did you know this?
1
Not at all
2
3
4
5
Perfectly
144
Q

What is the management of scalded skin?

A

IV Flucloxacillin

How well did you know this?
1
Not at all
2
3
4
5
Perfectly
145
Q

Withdrawal of steroids can be a trigger for what?

A

Psoriasis

How well did you know this?
1
Not at all
2
3
4
5
Perfectly
146
Q

Butterfly rash + ashleef macules + shagreen patch

A

Tuberous sclerosis

How well did you know this?
1
Not at all
2
3
4
5
Perfectly
147
Q

Management of molluscum

A

Watch and wait

How well did you know this?
1
Not at all
2
3
4
5
Perfectly
148
Q

Blistrs which cover most of the skin following an injury/graze?

A

Staph scalded skin

How well did you know this?
1
Not at all
2
3
4
5
Perfectly
149
Q

Skin prick vs patch testing?

A

Prick - T1 Hypersensitivity reactions
Patch - T4 Hypersensitivity

How well did you know this?
1
Not at all
2
3
4
5
Perfectly
150
Q

What is the most common type of BCC?

A

Nodular

How well did you know this?
1
Not at all
2
3
4
5
Perfectly
151
Q

Round/firm lesion following minor trauma?

A

Dermatofibroma

How well did you know this?
1
Not at all
2
3
4
5
Perfectly
152
Q

Hypopigmented skin on face/limbs/face with sensory loss in a traveller?

A

Leprosy -> treat with rifampicin, dapsone

How well did you know this?
1
Not at all
2
3
4
5
Perfectly
153
Q

Acne vulgaris is a disease of the…

A

pilosebaceous unit (blockage and inflammation)

How well did you know this?
1
Not at all
2
3
4
5
Perfectly
154
Q

Types of acne vulgaris lesions (several diff types usually seen in each pt)?

A
  • comedones eg. whitehead and blackhead
  • inflamm lesions eg. papules and pustules
  • nodules and cysts
  • ice-pick scars and hypertrophic scars
How well did you know this?
1
Not at all
2
3
4
5
Perfectly
155
Q

What are comedone lesions in acne vulgaris due to? When is it a white or blackhead?

A

Due to dilated sebaceous follicle.
Whitehead= if top is closed
Black= if top opens

How well did you know this?
1
Not at all
2
3
4
5
Perfectly
156
Q

Examples of inflam lesions in acne vulgaris?

A

papules, pustules

nodules and cysts (XS inflam response)

How well did you know this?
1
Not at all
2
3
4
5
Perfectly
157
Q

Inflammatory lesions in acne vulgaris eg. papules and pustules form when?

A

the follicle bursts releasing irritants

How well did you know this?
1
Not at all
2
3
4
5
Perfectly
158
Q

An excessive inflamm response in acne vulgaris may result in what

A

nodules and cysts

How well did you know this?
1
Not at all
2
3
4
5
Perfectly
159
Q

Due to the inflammatory lesions and response, what type of scars can be caused in acne vulgaris?

A

ice-pick and hypertrophic scars

How well did you know this?
1
Not at all
2
3
4
5
Perfectly
160
Q

What type of acne is often monomorphic?

A

drug-induced

How well did you know this?
1
Not at all
2
3
4
5
Perfectly
161
Q

example of drug induced acne?

A

pustules seen in steroid use

How well did you know this?
1
Not at all
2
3
4
5
Perfectly
162
Q

Acne fulminans?

A

V. severe acne associated with systemic upset eg. fever

How well did you know this?
1
Not at all
2
3
4
5
Perfectly
163
Q

Mx of acne fulminans?

A

hospital admission often needed and responds to oral steroids

How well did you know this?
1
Not at all
2
3
4
5
Perfectly
164
Q

Acne vulgaris is a chronic inflam skin condition affecting where

A

mainly face, back and chest

How well did you know this?
1
Not at all
2
3
4
5
Perfectly
165
Q

Cx of acne?

A

skin changes eg. scarring, post-inflam hyperpigmentation or depigmentation and pyschosocial problems

How well did you know this?
1
Not at all
2
3
4
5
Perfectly
166
Q

Advice to all people with acne?

A
  • avoid over-cleaning skin
  • use non-alkaline synthetic detergent cleansing product twice daily
  • avoid oil based skin care products, makeup and sun screens
  • persistent picking/scratch= scar
How well did you know this?
1
Not at all
2
3
4
5
Perfectly
167
Q

How long may treatments for acne vulgaris to be effective?

A

6-8w, may irritate skin esp at start of treatment

How well did you know this?
1
Not at all
2
3
4
5
Perfectly
168
Q

When to follow up pt with acne?

A

12w

How well did you know this?
1
Not at all
2
3
4
5
Perfectly
169
Q

Urgent referral for acne?

A

acne fulminans- same day dermatology to be assessed within 24hrs

How well did you know this?
1
Not at all
2
3
4
5
Perfectly
170
Q

When should referral to dermatology be considered in acne?

A
  • mild-moderate not responded to 2 completed courses of Mx
  • moderate-severe not responded to previous Mx that includes Abx
  • acne with scarring
  • acne with persistent pigmentary changes
  • causing psychological distress
How well did you know this?
1
Not at all
2
3
4
5
Perfectly
171
Q

Mx for mild to moderate acne?

A

12 week course of one of the 1st line options:

  • topical adapalene + topical benzoyl peroxide
  • topical tretinoin + topical clindamycin
  • topical benzoyl peroxide + topical clindamycin
How well did you know this?
1
Not at all
2
3
4
5
Perfectly
172
Q

How long is the course for acne Mx?

A

12w

How well did you know this?
1
Not at all
2
3
4
5
Perfectly
173
Q

Mx for moderate to severe acne?

A

12 week course of one of the 1st line options:

  • topical adapalene + topical benzoyl peroxide
  • topical tretinoin + topical clindamycin
  • topical adapalene + topical benzoyl peroxide + oral lymecycline 408mg or oral doxy 100mg OD
  • topical azelaic acid TDS + oral lymecycline or doxy
How well did you know this?
1
Not at all
2
3
4
5
Perfectly
174
Q

What can be used in acne Mx in combination to topical agents instead of systemic Abx in women?

A

COCP

How well did you know this?
1
Not at all
2
3
4
5
Perfectly
175
Q

Topical retinoids and oral tetracyclines are contradicted in

A

pregnancy and when planning pregnancy, and children <12.
Can use erythromycin instead

How well did you know this?
1
Not at all
2
3
4
5
Perfectly
176
Q

Mild acne vulgaris?

A

open and closed comedones with or without sparse inflam lesions

How well did you know this?
1
Not at all
2
3
4
5
Perfectly
177
Q

Moderate acne vulgaris?

A

widespread non-inflam lesions (comedones) and numerous papules and pustles (inflam)

How well did you know this?
1
Not at all
2
3
4
5
Perfectly
178
Q

Severe acne vulgaris?

A

Extensive inflam lesions, may include nodules (XS inflam lesions), pitting and scarring.

How well did you know this?
1
Not at all
2
3
4
5
Perfectly
179
Q

Only continue a Mx for acne that includes Abx (topical or oral) for more than 6m in

A

exceptional circumstances

How well did you know this?
1
Not at all
2
3
4
5
Perfectly
180
Q

Acne vulgaris is the obstruction of what

A

pilosebaceous follicle.
Follicular epidermal hyperproliferation results in formation of keratin plug that causes obstruction of follicle.

How well did you know this?
1
Not at all
2
3
4
5
Perfectly
181
Q

Pathophysiology of acne vulgaris

A
  • colonisation of bacteria
  • inflammation
  • obstruction of pilosebaceous follicle
How well did you know this?
1
Not at all
2
3
4
5
Perfectly
182
Q

Activity of sebaceous glands may be controlled by what?

A

androgen (but levels often normal in pts); so higher occurance in adolescence

How well did you know this?
1
Not at all
2
3
4
5
Perfectly
183
Q

When do 70-90% cases of atopic eczema occur?

A

<5yrs, most in 1st yr

How well did you know this?
1
Not at all
2
3
4
5
Perfectly
184
Q

Pattern of atopic eczema?

A

Chronic, itchy inflamm skin condition. Typically episodic with flares (exacerbations can be up to 2-3times a month) and remissions; can be continuous if severe.

How well did you know this?
1
Not at all
2
3
4
5
Perfectly
185
Q

Aeitology of atopic eczema?

A

genetic, skin barrier dysfunction, environmental factors (pets, house dust mites, pollen), atopic Hx, immune system dysfunction

How well did you know this?
1
Not at all
2
3
4
5
Perfectly
186
Q

Cx of atopic eczema?

A

infection eg. staoh aureus, HSV, superficial fungal.
psychological issues

How well did you know this?
1
Not at all
2
3
4
5
Perfectly
187
Q

Diagnosis for atopic eczema?

A

clinical: severity and impact

How well did you know this?
1
Not at all
2
3
4
5
Perfectly
188
Q

When to arrange immediate hospital admission for atopic eczema?

A

if eczema herpeticum is suspected (rapidly worsening, painful eczema, clustered blisters, punched out erosions)

How well did you know this?
1
Not at all
2
3
4
5
Perfectly
189
Q

When to refer to dematologist for atopic eczema?

A

uncertain, not controlled with current Mx, recurrent secondary infection, high risk of Cx, Mx advice needed

refer to paeds, derm or immuno if suspect food allergy trigger

refer to psych if controlled but QOL not improved

How well did you know this?
1
Not at all
2
3
4
5
Perfectly
190
Q

Mx for atopic eczema

A

stepwise approach

1) emollients and advice
2) topical corticosteroids eg. hydrocortisone 1%
3) antihistamine short course
4) short course oral corticosteroid
5) Abx treatment: flucloxacillin or clarithromycin

Occlusive dressings or dry bandages may be of benefit but only started by healthcare proff trained in their use/referral

How well did you know this?
1
Not at all
2
3
4
5
Perfectly
191
Q

Advice for atopic eczema?

A

measures to maintain skin and reduce risk of flare, self-care advice, avoid triggers

How well did you know this?
1
Not at all
2
3
4
5
Perfectly
192
Q

When are emollients used in Mx of atopic eczema?

A

first line during acute flares and remissions
Use frequent and liberally.
Large quantities prescribed= eg. 250g/week

How well did you know this?
1
Not at all
2
3
4
5
Perfectly
193
Q

When should topical steroids be used for atopic eczema?

A

2nd line
Hydrocortisone 1% (mild), betamthasone valerate 0.025% (moderate,severe)
Red, inflamed skin, lowest potency and amount of topical corticosteroid necessary.

How well did you know this?
1
Not at all
2
3
4
5
Perfectly
194
Q

How long should pt use topical corticosteroids for atopic eczema?

A

48hrs after the flare has been controlled, max 5 days on face.
Review at 3-6m if heavy usage.

How well did you know this?
1
Not at all
2
3
4
5
Perfectly
195
Q

Different potencys of topical corticosteroids for atopic eczema?

A

1) Mild & face= hydrocortisone 1%
2) Moderate= betamethasone valerate 0.025% (Betnovate RN)
3) Potent:Betamethasone valerate 0.1% (Betnovate)
4) Very potent= clobetasol propionate 0.05% (Dermovate)

DO NOT USE potent in <12m or very potent in any child

How well did you know this?
1
Not at all
2
3
4
5
Perfectly
196
Q

Preventative Mx for atopic eczema between flares?

A

Consider maintenance regimen of topical corticosteroids on prone areas but not face; consider step down approach or intermittent treatment

How well did you know this?
1
Not at all
2
3
4
5
Perfectly
197
Q

When should a non-sedating or sedating antihistamine be considered?

A

3rd line
1 month trial if persistent, severe itch or urticaria; sedating if affecting sleep

How well did you know this?
1
Not at all
2
3
4
5
Perfectly
198
Q

When should a short course of oral corticosteroids be used for Mx of atopic eczema?

A

4th line
If severe, extensive eczema

How well did you know this?
1
Not at all
2
3
4
5
Perfectly
199
Q

When should Abx treatment be prescribed in the Mx for atopic eczema?

A

If weeping, crusted or pustules, with fever or malaise, 2 bacterial infection should be considered

How well did you know this?
1
Not at all
2
3
4
5
Perfectly
200
Q

CP of atopic eczema?

A

general dryness and itching often localised to flexure of limbs

infants- invl fave, scalp and extensor surfaces, nappy area spared

acute flares of eczema= varies, poorly demarcated redness, fluid in skin (vesicles), scaling or crusting

chronic= thickened (lichenified) skin from repeated scratching; follicular hyperkeratotic papules (keratosis pilaris) may present on extensor surfaces of upper arms, buttocks and anterior thighs

How well did you know this?
1
Not at all
2
3
4
5
Perfectly
201
Q

Type of rash in atopic eczema?

A

itchy, erythoematous rash; scratching may exacerabate

How well did you know this?
1
Not at all
2
3
4
5
Perfectly
202
Q

Triggers for atopic eczema?

A
  • irritant allergens eg. detergents, soaps
  • irritant clothing eg. synthetic fabrics and wool (recommend cotton)
  • skin infections eg. s. aureus
  • contact allergens eg. perfume, latex, metals, topical meds
  • inhalant allergens eg. pollen, pets
  • hormonal triggers eg. premenstrual flares
  • climate eg. cold or sweating
  • concurrent illness & family life eg. stress, teething, lack of sleep
  • dietary eg. milk, wheat
How well did you know this?
1
Not at all
2
3
4
5
Perfectly
203
Q

Do pts with atopic eczema need allergy testing?

A

mostly no

How well did you know this?
1
Not at all
2
3
4
5
Perfectly
204
Q

Mild eczema?

A

areas of dry skin, and infrequent itching (with or without small areas of redness

How well did you know this?
1
Not at all
2
3
4
5
Perfectly
205
Q

Moderate eczema?

A

areas of dry skin, frequent itching, and redness (with or without excoriation and localized skin thickening)

How well did you know this?
1
Not at all
2
3
4
5
Perfectly
206
Q

Severe eczema?

A

widespread areas of dry skin, incessant itching, and redness (with or without excoriation, extensive skin thickening, bleeding, oozing, cracking, and alteration of pigmentation)

How well did you know this?
1
Not at all
2
3
4
5
Perfectly
207
Q

Infected eczema?

A

weeping, crusted, or there are pustules, with fever or malaise

How well did you know this?
1
Not at all
2
3
4
5
Perfectly
208
Q

Assess QOL for pt with eczema?

A

(0–10) of the person’s assessment of severity, itch, sleep loss, impact on everyday activities, psychosocial wellbeing over the last 3 days and nights

How well did you know this?
1
Not at all
2
3
4
5
Perfectly
209
Q

What is wet wrapping in atopic eczema?

A

Large amounts of emollients applied under wet bandages

How well did you know this?
1
Not at all
2
3
4
5
Perfectly
210
Q

In atopic eczema, how do u apply topical emollients and steroids?

A

Emollient first then wait 30mins before applying steroid.
Creams soak in faster than ointments

How well did you know this?
1
Not at all
2
3
4
5
Perfectly
211
Q

How much topical steroid to apply in eczema?

A

1 finger tip unit= 0.5g is sufficient to treat a skin area of twice that of flat adult hand

How well did you know this?
1
Not at all
2
3
4
5
Perfectly
212
Q

Most common type of cancer

A

BCC

How well did you know this?
1
Not at all
2
3
4
5
Perfectly
213
Q

How many types of skin cancer?

A

3

How well did you know this?
1
Not at all
2
3
4
5
Perfectly
214
Q

Basal cell carcinoma (BCC) lesions?

A

‘rodent’ ulcers and characterised by slow-growth and local invasion

How well did you know this?
1
Not at all
2
3
4
5
Perfectly
215
Q

Do BCC metastases

A

no, metastases extremely rare

How well did you know this?
1
Not at all
2
3
4
5
Perfectly
216
Q

Most common type of BCC?

A

nodular BCC

How well did you know this?
1
Not at all
2
3
4
5
Perfectly
217
Q

Where are BCC typically found?

A

sun-exposed sites eg. head and neck mainly

How well did you know this?
1
Not at all
2
3
4
5
Perfectly
218
Q

BCC CP?

A

initially a pearly, flesh-coloured papule with telangiectasia; may later ulcerate leaving a central ‘crater’

How well did you know this?
1
Not at all
2
3
4
5
Perfectly
219
Q

Referral for BCC?

A

if BCC suspected, routine referral

How well did you know this?
1
Not at all
2
3
4
5
Perfectly
220
Q

Mx options for BCC?

A

surgical removal, curettage, cryotherapy, radiotherapy, topical cream eg. imiquimod

How well did you know this?
1
Not at all
2
3
4
5
Perfectly
221
Q

Cellulitis?

A

acute bacterial infection of the dermis and subcutaneous tissue via disruptions in the cutaneous barrier

How well did you know this?
1
Not at all
2
3
4
5
Perfectly
222
Q

What areas most commonly affected in cellulitis?

A

lower limb

How well did you know this?
1
Not at all
2
3
4
5
Perfectly
223
Q

CP of cellulitis?

A
  • mainly lower limb, usually unilater
  • acute red, hot, swollen tender skin that spreads rapidly
  • diffuse border/ well-demarcated edge
  • blisters and bullae may form
  • fever, malaise, nausea and rigors may accompany or precede skin changes
  • may get lymphagitis
How well did you know this?
1
Not at all
2
3
4
5
Perfectly
224
Q

RFs for cellulitis?

A

skin trauma, ulceration, obesity

How well did you know this?
1
Not at all
2
3
4
5
Perfectly
225
Q

Cx of cellulitis?

A

necrotising fasciitis, sepsis, persistent leg ulceration, recurrent cellulitis

How well did you know this?
1
Not at all
2
3
4
5
Perfectly
226
Q

Is recurrence of cellulitis common?

A

yes and each episode increases likelihood of subsequent recurrence

How well did you know this?
1
Not at all
2
3
4
5
Perfectly
227
Q

Ix for cellulitis?

A
  • clinical usually: mark borders with pen to monitor progress
  • swab for culture if: penetrating injury, exposure to water-borne organisms, or if infection acquired outside the UK
How well did you know this?
1
Not at all
2
3
4
5
Perfectly
228
Q

Differentials of cellulitis?

A

DVT, septic arthritis, acute gout, ruptured Baker’s cyst

How well did you know this?
1
Not at all
2
3
4
5
Perfectly
229
Q

Most common causative organisms for cellulitis?

A

step pyogenes and staph aureus

How well did you know this?
1
Not at all
2
3
4
5
Perfectly
230
Q

What classification can be useful guide for assisting Mx for cellulitis?

A

Eron classification

How well did you know this?
1
Not at all
2
3
4
5
Perfectly
231
Q

Referral to hospital for cellulitis criteria?

A
  • CP of more serious condition
  • signs of systemic illness
  • have cormorbidity that may complicate or delay recovery or unstable
  • limb-threatening infection due to vascular compromise
  • severely immunocompromised
How well did you know this?
1
Not at all
2
3
4
5
Perfectly
232
Q

When should hospital admission for IV Abx be considered for cellulitis?

A
  • severely unwell, frail, immunocompromised, elderly or very young
  • facial cellulitis (unless v mild)
  • infection near eyes or nose incl. periorbital cellulitis
  • spreading infection not responding to oral Abx
  • lymphangitis
  • can’t take oral Abx
  • needed a swab if ?uncommon pathogen
How well did you know this?
1
Not at all
2
3
4
5
Perfectly
233
Q

Mx for cellulitis?

A
  • Abx
  • analgesia
  • advice: fluid intake, elevate leg to relieve oedema
  • preventative measures: weight loss, emollients to prevent dry skin and cracking
How well did you know this?
1
Not at all
2
3
4
5
Perfectly
234
Q

What cormorbitidies may cause cellulitis to spread rapidly or delay healing?

A

DM, PAD, chronic venous insufficiency, morbid obesity

How well did you know this?
1
Not at all
2
3
4
5
Perfectly
235
Q

Eron classification for cellulitis?

A

I= no signs systemic toxicity & no uncontrolled comorbidities

II= systemically well or unwell but with a cormorbitiy

III= systemic upset signif eg. confusion, tachycardia & pnoea, hypotension or unstable cormorbitity

IV= sepsis or life-threatening infection eg. necrotising fasciitis

How well did you know this?
1
Not at all
2
3
4
5
Perfectly
236
Q

When to reassess pt with cellulitis?

A

CP worsen or don’t improve 2-3 days; systemically unwell; pain out of proportion to infection; redness or swelling beyond inital presentation develops

How well did you know this?
1
Not at all
2
3
4
5
Perfectly
237
Q

When could prophylactic Abx be used in pt with cellulitis?

A

recurrent episodes (2 in <12m), consider routine referral for advice

How well did you know this?
1
Not at all
2
3
4
5
Perfectly
238
Q

Abx for cellulitis?

A

flucloxacillin 500-1000mg 4 times day for 5-7 days
or
clarithromycin 500mg twice day 5-7 days

eyes or nose= co-amoxiclav 500/125mg 3 times day 7days + specialist advice (or clarithromycin + metronidazole 400mg 3 times day)

How well did you know this?
1
Not at all
2
3
4
5
Perfectly
239
Q

Pediculus humanus capitis?

A

head lice

How well did you know this?
1
Not at all
2
3
4
5
Perfectly
240
Q

Head lice (Pediculus humanus capitis)?

A

parasitic insects that infest the hairs of human head and feed on blood from scalp, typically 30 lice per head

How well did you know this?
1
Not at all
2
3
4
5
Perfectly
241
Q

What if head lice (Pediculus humanus capitis) goes untreated?

A

may persist for long periods

How well did you know this?
1
Not at all
2
3
4
5
Perfectly
242
Q

How to confirm presence of head lice (Pediculus humanus capitis)?

A

detection combing= systematic combing of wet or dry hair using a fine toothed (0.2-0.3mm apart) head lice detection comb

How well did you know this?
1
Not at all
2
3
4
5
Perfectly
243
Q

Diagnosis for head lice (Pediculus humanus capitis)?

A

Only if live head louse is found on detection combing (not if just eggs or itchy scalp)

How well did you know this?
1
Not at all
2
3
4
5
Perfectly
244
Q

When to treat head lice (Pediculus humanus capitis)?

A

Only treat if live head louse found. All affected household members should be treated on same day (don’t treat if not affected)/

How well did you know this?
1
Not at all
2
3
4
5
Perfectly
245
Q

Mx for head lice (Pediculus humanus capitis)?

A

Either:
- physical insecticide eg. dimeticone 4% lotion (Hedrin)

  • traditional insecticide eg. malathion 0.5% aqueous liquid (Derbac-M)
  • Wet combing with fine-toothed head louse comb eg. Bug Buster comb

no treatment can guarantee success

How well did you know this?
1
Not at all
2
3
4
5
Perfectly
246
Q

What to do after treating head lice (Pediculus humanus capitis)?

A

detection combing again to confirm success

How well did you know this?
1
Not at all
2
3
4
5
Perfectly
247
Q

What if Mx of head lice (Pediculus humanus capitis) is unsuccessful?

A
  • ?used correctly
  • identify source of reinfestation by assessing household members, close family and friends
  • repeat Mx (possible resistance of malathion)
How well did you know this?
1
Not at all
2
3
4
5
Perfectly
248
Q

Advice for head lice (Pediculus humanus capitis)?

A
  • can still go to school
  • no preference for clean or dirty hair
  • no need to treat bedding or clothing as lifespan when detached from human head is 1-2days
  • not possible to prevent so if primary school age examine regularly at home
How well did you know this?
1
Not at all
2
3
4
5
Perfectly
249
Q

Nits vs head lice?

A

Head lice= tiny insects that live in hair
Nits= empty egg cases attached to hair that head lice hatch from

How well did you know this?
1
Not at all
2
3
4
5
Perfectly
250
Q

Impetigo?

A

common superficial highly contagious bacterial skin infection, most common in children

How well did you know this?
1
Not at all
2
3
4
5
Perfectly
251
Q

Types of impetigo?

A

Non-bullous (70%) and bullous

How well did you know this?
1
Not at all
2
3
4
5
Perfectly
252
Q

Causes of non-bullous impetigo?

A

staph aureus, strep pyogenes or both

MRSA becoming more common

How well did you know this?
1
Not at all
2
3
4
5
Perfectly
253
Q

Causes of bullous impetigo?

A

staph aureus

MRSA becoming more common

How well did you know this?
1
Not at all
2
3
4
5
Perfectly
254
Q

RFs for impetigo?

A

conditions that leads to breaks in skin eg. cuts, burns, bites, eczema, contact dermatitis; warm/humid weather; poor hygiene; crowded environments

How well did you know this?
1
Not at all
2
3
4
5
Perfectly
255
Q

Do you need treatment for impetigo?

A

Usually self-limiting, heals within 7-21 days without treatment. But Abx can lead to quicker resolution and reduced infective period.

How well did you know this?
1
Not at all
2
3
4
5
Perfectly
256
Q

Cx of impetigo?

A

RARE
eg. glomerulonephritis, cellulitis
can occur in neonates and severe immunosupressed

How well did you know this?
1
Not at all
2
3
4
5
Perfectly
257
Q

Ix for impetigo?

A
  • clinical
  • swabs for culture and sensitivities considered if persistent, recurrent or widespread
How well did you know this?
1
Not at all
2
3
4
5
Perfectly
258
Q

Differentials for impetigo?

A

chicken pox, eczema, cellulitis

How well did you know this?
1
Not at all
2
3
4
5
Perfectly
259
Q

CP for non-bullous impetigo?

A

thin-walled vesicles/pustules that rupture quickly forming GOLDEN brown crusts. Most commonly the face (around nose and mouth), limbs and flexures eg. axillae

How well did you know this?
1
Not at all
2
3
4
5
Perfectly
260
Q

Cp for bullous impetigo?

A

large fragile flaccid bullae (fluid filled lesions) that rupture and ooze yellow fluid, leaving a scaley rim (collarette). Affects face, flexures, trunk and limbs.

How well did you know this?
1
Not at all
2
3
4
5
Perfectly
261
Q

Mx of impetigo?

A
  • advice on hygiene measures
  • advice on exclusion
  • short course hydrogen peroxide 1% cream, topical Abx (fusidic acid 2%) or oral Abx depending on severity
  • reassess if rapidly worsen or not improved once completed course
How well did you know this?
1
Not at all
2
3
4
5
Perfectly
262
Q

Exclusion rules for impetigo?

A

not attend school/work ect. until all lesions are healed, dry and crusted over or until 48hrs after commencing treatment

How well did you know this?
1
Not at all
2
3
4
5
Perfectly
263
Q

When would pt with impetigo need admitted, referral or specialist advice?

A
  • admit if ?sepsis
  • refer: immunosuppressed and infection widespread
  • microbio consult if ?MRSA
How well did you know this?
1
Not at all
2
3
4
5
Perfectly
264
Q

Do you typically get systemic symptoms eg. fever with impetigo?

A

no

How well did you know this?
1
Not at all
2
3
4
5
Perfectly
265
Q

Hygiene measures to aid healing and reduce spread of impetigo?

A

wash affected areas with soap and water; avoid touching the impetigo; wash hands regularly; cover affected areas where possible; don’t share towels; wash clothes and beddings on hottest setting daily during first few days of Mx

How well did you know this?
1
Not at all
2
3
4
5
Perfectly
266
Q

If you consider treatment for impetigo, what should be used if it is localised?

A

hydrogen peroxide 1% cream 2-3 times daily 5 days

How well did you know this?
1
Not at all
2
3
4
5
Perfectly
267
Q

If you consider treatment for impetigo, what should be used if it is widespread?

A

Topical fusidic acid 2 % 3x daily for 5 days
If resistant use mupirocin 2%

How well did you know this?
1
Not at all
2
3
4
5
Perfectly
268
Q

If you consider treatment for impetigo, what should be used if localised or widespread and systemically unwell?

A

Oral Abx

> 18yrs= flucloxacillin 500mg 4x day for 5 days

10-17= 250-500mg

2-9= 125-250mg

1m-1yr= 62.5-125mg

Allergic= clarithromycin 250mg 2x day 5d
Pregnant= eryhromycin 250-500mg 4x day 5d

How well did you know this?
1
Not at all
2
3
4
5
Perfectly
269
Q

Melanoma?

A

malignant tumour arising from melanocytes in the skin

How well did you know this?
1
Not at all
2
3
4
5
Perfectly
270
Q

4 common subtypes of melanoma?

A
  • superficial spreading
  • nodular
  • lentigo maligna
  • acral lentiginous
How well did you know this?
1
Not at all
2
3
4
5
Perfectly
271
Q

Melanoma: other types of pigmented lesions?

A

dermatofibromas, freckles, lentigines, moles (naevi), pigmented BCC, seborrhoeic keratoses

How well did you know this?
1
Not at all
2
3
4
5
Perfectly
272
Q

Ix for melanoma:

A
  • clinical exam: use 7 point checklist to assess pigmented skin lesions and entire skin surface
How well did you know this?
1
Not at all
2
3
4
5
Perfectly
273
Q

Urgent 2ww referral should be arranged for people with a pigmented lesion suspicious for melanoma when?

A
  • lesion scored 3 or more on checklist
  • dermoscopy suggests melanoma
  • nail changes eg. new pigmented line in nail or lesion growing under nail
  • new persistent skin condition esp if growing, pigmented or vascular in appearance
  • there is any doubt
  • biopsy has confirmed melanoma
How well did you know this?
1
Not at all
2
3
4
5
Perfectly
274
Q

What to ask about when taking a history for melanoma?

A
  • when lesion was 1st noticed
  • has it changes in size, shape or colour
  • if it has ulcerated
  • if it is itchy or has bled
  • cough, weight loss, fatigue ect
How well did you know this?
1
Not at all
2
3
4
5
Perfectly
275
Q

How to examine a lesion in suspected melanoma?

A
  • good light, with or without magnification
  • dermatoscope if trained
  • examine entire surface of skin incl. scalp and mucous membranes: palpate major lymph nodes
  • use 7 point checklist
How well did you know this?
1
Not at all
2
3
4
5
Perfectly
276
Q

What type of lesions should raise suspicion if ?melanoma?

A

atypical melanocytic lesions that are different from person’s surrounding moles (‘Ugly Duckling sign’)

How well did you know this?
1
Not at all
2
3
4
5
Perfectly
277
Q

7 point checklist for assessment of pigmented skin lesions and to determine need for referral (?melanoma)?

A

Major features (2points each):
- change in size
- irregular shape
- irregular colour

Minor features (1 point each):
- largest diameter 7mm or more
- inflammation
- oozing
- change in sensation (incl itch)

How well did you know this?
1
Not at all
2
3
4
5
Perfectly
278
Q

Should you biopsy in primary care if melanoma is suspected or diagnosis is uncertain?

A

no, refer for 2ww

How well did you know this?
1
Not at all
2
3
4
5
Perfectly
279
Q

Consider 2ww referral for melanoma?

A

pigmented or non-pigmented skin lesion that suggests nodular melanoma or any major feature on the 7 point checklist

How well did you know this?
1
Not at all
2
3
4
5
Perfectly
280
Q

Who should you routinely refer for suspected melanoma?

A

pt at greatly increased risk of melanoma eg.
- giant congenital pigmented hairy naevi
- FHx of 3+ cases of melanoma

How well did you know this?
1
Not at all
2
3
4
5
Perfectly
281
Q

Advice on prevention of melanoma?

A

avoid sunburn, sunscreen with at SPF min 15, avoid sunbeds, vit D supplements

How well did you know this?
1
Not at all
2
3
4
5
Perfectly
282
Q

Advice for pts to seek medical advice if they have a mole with what features?

A

A.symmetrical
B.orders irregular
C.olour uneven
D.iameter >6mm wide
E.volving: changing size, shape or colour

Itchy, painful, bleeding, crust, inflammed

How well did you know this?
1
Not at all
2
3
4
5
Perfectly
283
Q

Most aggressive form of melanoma?

A

nodular, others spread more slowly

How well did you know this?
1
Not at all
2
3
4
5
Perfectly
284
Q

Superficial spreading melanoma CP? (70%)

A

typically affects= arms, legs, chest, back; young people

a growing mole with diagnostic features

285
Q

Nodular melanoma CP? (2nd most common)

A

affects= sun exposed skin; middle-aged

red or black lump or lesion which bleeds or oozes

286
Q

Lentigo maligna melanoma CP?

A

affects= chronically sun-exposed skin; older people

growing mole with diagnostic features

287
Q

Acral lentiginous melanoma CP?

A

affects= nails, palms or soles; people with darker skin pigmentation

subungal hyperpigmentation (Hutchinson’s sign) on palms or feet

288
Q

Mx for melanoma

A
  • Suspicous lesions= excision biopsy
  • Once diagnosis confirmed: determine whether further re-excision of margins is required
  • Further Mx selectively= sentinel lymph node mapping, isolated limb perfusion and block dissection of regional lymph node groups

If doubt about lesion, avoid excision in primary care and refer.

289
Q

Margins of excision of melanoma related to Breslow thickness?

A
  • 0-1mm thick= 1cm exicison
  • 1-2mm= 1-2cm (depends on site and pathological features
  • 2-4mm= 2-3cm
  • > 4mm= 3cm
290
Q

Approx 5yr survival according to Breslow thickness of melanoma?

A

<0.75mm= 95-100%

0.76-1.50mm= 80-96%

1.51-4mm= 60-75%

> 4mm= 50%

291
Q

Psoriasis?

A

systemic, immune-mediated, inflamm skin disease which typically has a chronic relapsing-remitting course, and may have nail and joint (psoriatic arthritis) invl.

292
Q

Most common form of psoriasis? (80%)

A

chronic plaque psoriases (incl. scalp, flexural and facial psoriasis)

293
Q

Forms of psoriasis?

A
  • chronic plaque P (most common)
  • localised pustular psoriasis of palms and soles (2nd most common)
  • guttate P
  • nail P
  • erythrodermic and generalised pustular P
294
Q

what form of psoriasis is a rare medical emergency and may be life-threatening?

A

erythrodermic and generalised pustular psoriasis

295
Q

Factors associated with the onset or exacerbation of psoriasis?

A

infection, drugs (incl. corticosteroid withdrawl), UV exposure, trauma, hormonal changes, stress, smoking, alcohol

296
Q

Psoriasis may be associated with what other conditions?

A

psoriatic arthritis, metabolic syndrome, IBD (crohns), anxiety, depression

297
Q

Ix for psoriasis

A

Usually clinical: assess CP and classify the type of psoriasis. Assess the proportion of total body surface area (BSA) affected. Any systemic illness?

298
Q

CP of psoriasis?

A
  • distribution=often on extensor surfaces (elbows and knees), tunk, flexures, sacral and natal cleft, scalp and behind ears, umbilicus
  • size and shape of lesions= usually clear delineation between normal and affected skin
  • colour= pink or red, if pigmented skin may not be obvious; scale is silvery in colour
  • invl of other area= joints or nails
299
Q

What does Mx of psoriasis depend on?

A

type, site, extent of involvement, impact on life

300
Q

Mx for psoriasis may include what?

A
  • lifestyle advice
  • Mx of stress, anxiety ect
  • Topical preparations
  • Specialist treatments
301
Q

Give pt advice on what if they have psoriasis, particularly if severe?

A
  • assess CVD risk at least every 5yrs
  • reduce risk of VTE
  • seek urgent medical advice if unexplained joint pain, swelling
302
Q

Possible topical preparations for Mx of psoriasis?

A
  • emollients
  • corticosteroids
  • vit D analogues
  • coal tar
  • short-contact dithranol (for large plaque psoriasis)

depends on preference, cosmetic acceptability, practicality

303
Q

Possible specialist treatments for psoriasis?

A
  • topical calcineurin inhibitors
  • phototherapy
  • systemic or biologic therapy
304
Q

Referral to derm in psoriasis criteria?

A
  • uncertainty
  • extensive eg. >10% body surface affected
  • moderatley severe
  • resistant to topical drug treatments in primary care
  • nail disease that is severe and having major functional/cosmetic impact
  • impact on QOL
305
Q

What screening tool can be used for psoriasis?

A

Psoriasis Epidemiology Screening Tool, 3+/5 then consider referral to rheumatology

306
Q

How to use pts CP to classify their type of psoriasis?

A
  • distribution
  • size and shape of lesions
  • number of lesions
  • severity of lesions: 6-point Static Physician’s Global Assessment (PGA) score
  • surface features: smooth, scaly or pustular
  • colour
  • Auspitz sign
  • Invl of other areas
307
Q

How to estimate the totaly body surface area affected by psoriasis?

A

Rule of Nines (usually used for burns)

  • arm: 9%
  • head: 9%
  • neck: 1%
  • leg: 18%
  • anterior trunk: 18%
  • posterior trunk: 18%
308
Q

Types of pustular psoriasis?

A

Generalised or localised

309
Q

Generalised pustular psoriasis CP?

A

potentially life-threatening emergency

rapidly developing widespread erythema, followed by eruption of white, sterile non-follicular pustules which coalesce to form large lakes of pus

associated with systemic illness: fever, malaise, tachy, weight loss, arthralgia

usually in pt with existing or previous chronic plaque psoriasis

310
Q

Localised (palmoplantar) pustular psoriasis CP?

A

lesions on palms and soles

yellow-brown pustules within established psoriasis plaques or redness, scaling and pustules at tips of fingers and toes

311
Q

Most severe psoriatic disease?

A

Pustular psoriasis

312
Q

Pustular psoriasis?

A

orphan skin and multisystemic inflam disease characterised by intermittent flares or attacks with partial or complete remission between episodes

313
Q

Erythrodermic psoriasis CP?

A

potentially life-threatening emergency

diffuse widepread severe psoriasis affecting >90% body

can develop gradually from chronic plaque psoriasis or appear abruptly even in mild psoriasis

can be precipitated by systemic infection, irritatns eg. ciclosporin, coal tar, phototherapy or sudden withdrawl corticosteroids

lesions may feel warm and be associated with systemic illness: fever, malaise, tachy, lymphadenopathy, peripheral oedema

314
Q

What types of psoriasis are potentially life-threatening medical emergency?

A

erythrodermic &
pustular

315
Q

Chronic plaque psoriasis CP?

A

monomorphic erythematous plaques covered by adherent silvery-white scale, usually on scalp, behind ears, trunk, buttocks, periumbillical area and extensor surfaces (forearms, skins, elbows, knees)

316
Q

Type of lesions in chronic plaque psoriasis?

A

typically symmetrical, can coalesce to form larger lesions.
Most 1cm to several in diammeter, oval or irreglar shape
White skin= plaques red or pink
Pigmented skin= grey plaques
- clear delination between normal and affected
- scale usually present: usually silver-white, can be waxy yellow or orange, can be very thick
- Auspitz’s sign
- occasionally Woronoff’s ring

317
Q

Auspitz’s sign in psoriasis?

A

if scale gently removed, a glossy membrane with pinpoint bleeding points is revealed

318
Q

Woronoff’s ring in psoriasis?

A

halo-like effect seen around a plaque due to vasoscontriction

319
Q

Scalp psoriasis CP?

A

75-90% pts with psoriasis

presents as chronic psoriasis affecting scalp

whole scalp can be affected, or individual plaques may be visible, plaques may be v thick esp in occipital regio

320
Q

What may scalp psoriasis be associated with?

A

areas of non-scarring alopecia, particulary if there is thick adherent scale extending up to hair shaft (pityriasis amiantacea); erythrodermic psoriasis or repeated scratching of scalp due to itch (usually reversible)

321
Q

Facial psoriasis CP?

A

well-demarcated plaques on face, similar to chronic plaque psoriasis

lesions may affect hairline

possible mild scaling around eyebrows and nasolabial folds that may be due to co-existant seborrhoeic dermatitis (‘sebo-psoriasis)

322
Q

Flexural psoriasis CP?

A
  • itchy lesions affecting groin, genital area, axillae, inframammary folds, abdo folds, sacral and gluteal cleft

elederly, immobile, obese at higher risk

lesions of chronic plaque psoriasis that are well defined but may be little or no scaling due to friction and occlusion at these sites

lesions often red and glazed and may be fissure in skin crease

323
Q

Guttate psoriasis CP?

A

small, scattered, round or oval (2mm-1cm in diameter- water drop appearance) scaly papules which may be pink or red

multiple lesions may occur all over body over 1-7days

may affect trunk, proximal limbs; face, ears and scalep

mostly in children and young adults

324
Q

First presentation of guttate psoriasis?

A

after acute strep URTI (2-4w prior to lesions) or as acute exacerbation of plaque psoriasis

325
Q

Nail psoriasis CP?

A

commonly fingernails (50%) than toes (50%), may affect all parts of nail and surrounding structures

nail pitting, discolouration (oil drop sign; yellow-orange); subungual hyperkeratosis; onycholysis; complete nail dystrophy

326
Q

Nail changes can occur with what type of psoriasis?

A

Any, in particular psoriatic arthritis; increases during duration of psoriasis and less common in children

327
Q

Mx for pustular or erythrodermic psoriasis?

A

same day immediate dermatology assessment and ongoing Mx, medical emergency

328
Q

Advice for pt with chronic plaque psoriasis?

A
  • control symptoms, no cure
  • complete clearance of skin may not be possible
  • not infectious condition
  • weight loss, smoking cessation, alcohol within limits
  • stress and anxiety Mx
  • urgent medical advice if unexplained joint pain or swelling
329
Q

Mx for chronic plaque psoriasis of trunk and/or limbs?

A

Topical preparations: emollients to reduce scale and itch

1st= Patent topical corticosteroids od + topical vit D analogue od (one in morning, other in evening) for up to 4w as inital treatment

2nd= no improvement after 8w offer vit D analogue twice daily

3rd= no improvement after 8-12w offer potent corticosteroid twice daily up to 4 weeks or a coal tar preparation applied once or twice daily

4th= short-acting dithranol

5th= Consider if referral if meet criteria

330
Q

Rules for topical corticosteroids for psoriasis?

A

ONLY for localised areas

up to 4w as inital treatment
- od, stop once clear, no longer than 8w at one site, can be restarted after 4 week treatment break

331
Q

Topical preparations for chronic plaque psoriasis Mx?

A

(emollient reduces scale and relieves itch)
- creams, lotions or gels for widespread
- ointments for thick scale areas
- lotions, solutions or gels for hair

332
Q

How long for topical preparations to start to work for psoriasis Mx?

A

may take several weeks and if stopped suddenly there’s risk of relapse

333
Q

When to review pt with psoriasis?

A

4w after starting treatment and annually if using topical corticosteroids

334
Q

Specialist therapy for psoriasis?

A

1st= topical calcineurin inhibitors eg. tacrolimus

2nd= narrow-band ultraviolet UVB phototherapy 2-3x a week

3rd= systemic theraoy eg. methotrexate

4th= biologic therapy- TNF-alpha inhibitors eg. adalimumab sub cut or infliximab IV

335
Q

Mx for guttate psoriasis?

A

Reassure it’s self-limiting and typically resolves within 3-4m of onset

336
Q

Mx for nail psoriais?

A
  • advise keep nail short
  • if mild then no treatment is needed; nail varnish to disguise pitting but avoid acetone remover
  • if severe then specialist Mx
337
Q

Scalp psoriasis Mx?

A

potent topical cortiosteroids od for 4we

if no improvement after 4w, use different formulation of the potent corticosteroid eg. shampoo or mousse +/or topical agents to remove adherent scale before applying steroid eg. agents containing salicylic acid, emollients and oils

338
Q

Face, flexural and genital psoriasis Mx?

A

mild or moderate potency corticosteroid once or twice daily for max 2w

339
Q

Side effects of topical corticosteroid therapy eg. in psoriasis?

A

skin atrophy, striae and rebound symptoms. Face, scalp, flexures prone to steroid atrophy for do n’t use for >1-2w per month

systemic S/Es may be seen when used >10% body surface area

340
Q

Vitamin D analogues eg. for psoriasis example?

A

Calcipotriol (Dovonex) and calcitriol. Max is 100g a week, AVOID in pregnancy

341
Q

Vitamin D analogues eg. for psoriasis mechanism of action?

A

↓ cell division and differentiation → ↓ epidermal proliferation

342
Q

Why are vit D analogues for eg. psoriasis good?

A

adverse effects uncommon; can be used long term unlike corticosteroids; reduce scale and thickness of plaques but not erythema

343
Q

Dithranol for psoriasis?

A
  • inhibits DNA synthesis
  • wash off after 30mins
  • adverse effects= burning, staining
344
Q

Coal tar for psoriasis?

A

MOA not understood but probably inhibit DNA synthesis

345
Q

Tear drop papules on trunk and limbs, pink scaly patches or plaques, acute, strep infection 2-4w prior to lesions appearing?

A

Guttate psoriasis

346
Q

Guttate psoriasis vs Pityriasis rosea?

A

G= preceded by strep sore throat 2-4w
PR= many report recent resp infections but not common in questions

G= tear drop scaly papules on trunk and limbs
PR= herald patch followed 1-2w later by multiple erythematous slightly raised oval lesions with fine scale confined to outer lesions. Longitudinal diameters of oval lesions may run parrallel to line of Langer (produce fir-tree appearance)

G= most resolve spontaneously within 2-3m; topical agents as per psoriasis; UVB phototheraoy
PR= self-limiting, resolves after around 6w

347
Q

Psoriasis pathophysiology?

A

multifactorial
- genetic
- immunological
- environmental

348
Q

Psoriasis genetic associations?

A

HLA-B13, -B17 and -Cw6. 70% concordance in identical twins

349
Q

Psoriasis immunological association?

A

abnormal T cell activity stimulates keratinocyte proliferation

350
Q

Psoriasis environmental association?

A
  • worsened by skin trauma and stress
  • triggered eg. by strep infection
  • improved by sunlight
351
Q

Short summary of plaque psoriasis?

A

most common subtype resulting in typical well-demaracted red, scaly patches affecting extensor surfaces, sacrum and scalp

352
Q

Short summary of flexural psoriasis?

A

in contrast to plaque psoriasis the skin is smooth

353
Q

Short summary of guttate psoriasis?

A

transient psortiatic rash frequently triggered by trep infection; multiple teardrop lesions appear on body

354
Q

Short summary of pustular psoriaisis?

A

common on palms and soles

355
Q

2 other features of psoriasis?

A

arthritis and nail signs (pitting, onycholysis)

356
Q

Cx of psoriais?

A
  • psoriatic arthropathy (10%)
  • increased incidence of: metabolic syndrome, CVD, VTW
  • psychological distress
357
Q

What factors may exacerbate psoriasis?

A

trauma, alcohol, drugs, withdrawl of systemic corticosteroids

358
Q

drugs that may exacerbate psoriasis?

A

BB, lithium, antimalarials (cloroquine & hydroxychloroquine), NSAIDs, ACEi, infliximab

359
Q

Pts with psoriasis are at increased risk of what?

A

CVD, HTN, VTE, depression, UC & Crohn’s, non-melanoma skin cancer, other ca incl. liver, lung upper GI

360
Q

Scabies?

A

intensely itchy skin infection

Classical (typical) scabies involvesinfestation with a low number of mites (5-15 per host)

361
Q

What human parasite causes scabies?

A

Sarcoptes scabiei

362
Q

What is crusted (Norweigian) scabies?

A

hyperinfestation with thousands or millions of mites present in exfoliating scales of skin

363
Q

RFs for scabies?

A

close contact; poverty and social deprivation; living in crowded areas and institutionalisation; winter

364
Q

Who is crusted scabies primarily seen in?

A

Hx of immunosupression eg. HIV or long-term corticosteroids; pts with reduced ability to scratch eg. incapibility or not perceived due to anaesthesia; learning difficulites or neuro disorders; elderly

365
Q

Cx of scabies?

A

secondary bacterial infection -> cellulitis, folliculitis, boils, impetigo, lymphangitis

366
Q

Prognosis of scabies?

A

Classic= good if compliant with treatment and all close contacts treated
Crusted= prolonged or repeated treatment

367
Q

Ix for scabies?

A
  • clinical
  • confirm= skin scraping microscopy or ink borrow test
368
Q

Mx of scabies?

A
  • Topical insecticide (permethrin 5% cream)
  • Also treat all household members, close contacts and sexual partners even if asymptomatic
  • Topical crotamiton for symptomatic treatment of itching
  • Resistant scabies= oral ivermectin
369
Q

Is follow up required for scabies?

A

No unless not cleared 2-4w following Mx

370
Q

What is a child under 2m gets scabies?

A

Rare so seek specialist advice from paed derm

371
Q

Why should specialist advice be sought for crusted scabies?

A
  • may need hospital admission
  • may need combination therapy with topical insecticide + oral ivermectin
  • may need to Ix underlying immunodef.
372
Q

CP of scabies?

A
  • intense generalised itch worse at night
  • symptoms 3-6 after primary infestation but can occur 1-3d in reinfested person
  • erythematous papules
  • papules small and often excoriated with haemorrhagic crusts on top
  • burrow (pathognomonic sign) appears as thin bowrn grey line 0.2-1cm in length
  • Nodular lesions may be seen esp on penis and scrotum in men and persist after Mx (indicative of sexually acquired scabies)
373
Q

When does pt with scabies get symptoms?

A

symptoms 3-6 after primary infestation but can occur 1-3d in reinfested person

374
Q

What areas of the body are affected by scabies?

A
  • distribution on the periumbilical area, waist, genitalia, breasts, buttocks, axillary folds, fingers, wrists and extensor aspects of limbs
  • back not invl and head spared except in children
  • palms and soles affected in elderly, infants and young children
375
Q

Crusted scabies CP?

A
  • pruritus mild or absent due to impaired immune response
  • skin lesions: generalised, poorly defined, erythematous, fissured plaques covered by scales and crusts
  • may get diffuse non-crusted scabies with invl of back
  • bacterial 2 infection can lead to malodorous skin lesions
376
Q

What do plaques in crusted scabies look like on bony prominences eg. finger articulations, elbows and iliac crest?

A

yellow-to-brown thick verrucous aspect

377
Q

What are burrows (a pathognomonic sign of scabies) produced by?

A

moving mites, difficult to see if skin been scratched or infected

378
Q

What % of pts with SCC get mets?

A

2-5%

379
Q

RFs for SCC?

A
  • XS sunlight exposure/psoralen UVA therapy
  • actinic keratoses and Bowen’s
  • smoking
  • immunosupression eg. HIV, following renal transplant
  • long-standing leg ulcers (Majolin’s ulcer)
  • genetic conditions eg. xeroderma pigmentosum, occulocutaneous albinism
380
Q

Features of SCC?

A
  • typically sun-exposed sites: head, neck, dorsum of hands and arms
  • rapidly expanding painless ulcerate nodules
  • may have cauliflower-like appearance
  • may be areas of bleeding
381
Q

Mx for SCC?

A

surgical excision with 4mm margins if lesion <20mm diameter; if >20mm then 6mm margins

Mohs micrographic surgery may be used in high risk pts and cosmetically important sites

382
Q

Good prognosis of SCC?

A

well differentiated tumour; <20mm diameter; <2mm deep; no associated diseases

383
Q

Poor prognosis for SCC?

A

poorly differentiated tumours; >20mm diameter; >4mm deep; immunosupression

384
Q

Tinea?

A

term given to dermatophyte fungal infections

385
Q

Types of tinea infections?

A
  • tinea capitis= scalp
  • tinea corporis= trink, legs or arms
  • tinea pedis= feet
386
Q

Another name for tinea capitis?

A

scalp ringworm

387
Q

What is tinea capitis (scalp ringworm)?

A

cause of scarring alopecia mainly seen in children

388
Q

What is tinea capitis (scalp ringworm) is left untreated?

A

a rasied pustular spongy/boggy mass called a kerion may form

389
Q

Most common cause of tinea capitis (scalp ringworm)?

A

Trichophython tonsurans.

Another cause: Microsporum canis from cats or dogs

390
Q

Diagnosis of tinea capitis (scalp ringworm)?

A

Scalp scraping: skin and hair sampling for fungal microscopy and culture

391
Q

How to identify between the 2 different causes of tinea capitis (scalp ringworm)?

A

Lesions due to Microsporum canis green fluorescence under Wood’s lamp (Tichophyton tonsurans dont’)

392
Q

Mx for tinea capitis (scalp ringworm)?

A

Oral antifungals

AND

Topical ketoconazole shampoo twice weekly for 2-4w to reduce transmission

393
Q

Advice to give pt with tinea capitis (scalp ringworm)?

A

surface crust removal; discard or disinfect objects that can transmit infection; inspect family member or household pets for sign of infection

394
Q

How do you know what oral antifungals to give pt with tinea capitis (scalp ringworm)?

A

-Terbinafine for 4w= Trichophyton tonsurans infections, eg. live in urban area

  • Griseofulvin for 4-8w= Microsporum infections, eg. live in rural area

Start in adults before or after diagnosis confirmed depedning on clinical judgement; in children wait until confirmed or specialist advice

395
Q

What do you do once Mx has been initiated for pt with tinea capitis (scalp ringworm)?

A

review 4-8w after completed the course; can be continued for a further 2-4w

396
Q

When should referral to derm be arranged in pt with tinea capitis (scalp ringworm)?

A
  • has suspected kerion (urgent referral)
  • oral antifungal considered for child and lack of expertise to start in primary care
  • uncertain
  • Mx unsuccessful
397
Q

Another name for tinea corporis?

A

Ringworm- fungal skin infection of the body and groin

398
Q

What to do if a person is a known contact of a person with tinea capitits (scalp ringworm)?

A

skin and hair sampling for fungal culture should be arranged to see if confirmed infection or asymptomatic carrier

399
Q

Causes of tinea corporis (ringworm)?

A

Trichophyton rubrum and Trichophyton verrucosum (contact with cattle)

400
Q

RFs for tinea corporis (ringworm)?

A

hot, humid environments; tight fitting clothing; obesity; hyperhidrosis

401
Q

Ix for tinea corporis (ringworm)?

A

1st= clinical
2nd= skin sampling for fungal microscopy and culture is severe or uncertain

402
Q

Mx for tinea corporis (ringworm)?

A
  • advice on self care
  • Mild= topical antifungal cream eg. terbinafine
  • Marked inflam= short term midly potent topical corticosteoird eg. hydrocortisone cream
  • Severe= oral antifungal eg. terbinafine
403
Q

When should referral to derm specialist be arranged for pt with tinea corporis (ringworm)?

A
  • severe or topical antifungal unsuccessful in child
  • uncertain
  • Mx unsuccessful or frequent recurrences
  • immunocompromised
404
Q

Another name for Tinea pedis?

A

Athlete’s foot

405
Q

CP of tinea pedis (athlete’s foot)?

A

itchy and peeling skin between the toes, common in adolescence

406
Q

CP of tinea capitis (scalp ringworm)?

A

-scaling and itch of scalp; patches of hair loss; skin erythema; pustules; crusting; lymphadenopathy
- kerion= painful pustular boggy masses which may have thick crust

407
Q

CP of tinea corporis (ringwowm)?

A
  • scaly, itchy skin.
  • single/multiple red or pink, flat/slightly raised annular (ring-shaped) patches of varying sizes which enlarge outwards on the body.
  • lesions= active red, scaly advancing edge and a clear central area.
  • may be groin involvement of the inguinal folds and proximal medial thighs
408
Q

What is tinea pedis (athelete’s foot) caused by?

A

dermatophytes

409
Q

Sub-types of tinea pedis (athelete’s foot)?

A
  • interdigital (common)
  • Moccasin or dry
  • Vesicobullous (least common)
410
Q

Tinea pedis (athelete’s foot): Interdigital?

A

lateral toes web spaces first and cause by Trichophyton rubrum

411
Q

Tinea pedis (athelete’s foot): Moccasin or dry?

A

diffuse chronic scaling and hyperkeratosis affecting sole and lateral foot; caused by Trichophyton rubrum

412
Q

Tinea pedis (athelete’s foot): vesicobullous?

A

multiple small vesicles and blisters mainly on arches and soles of feet; caused by Trichophyton interdigitale

413
Q

RFs for tinea pedis (athelete’s foot)?

A

hot, humid environment; occlusive footwear; hyperhidrosis; walking on contaminated surfaces; immunocompromised

414
Q

Is infection common in tinea pedis (athelete’s foot)?

A

yes in adolescents but rare in pre-pubertal children

415
Q

Diagnosis of tinea pedis (athelete’s foot)?

A

1st= clinical
2nd= skin sampling for microscopy and culture if severe or uncertain

416
Q

Cp for tinea pedis (athelete’s foot)?

A
417
Q

Mx for tinea pedis (athelete’s foot)?

A
  • advice on self care
  • Mild= topical antifungal cream eg. terbinafine
  • Marked inflam= hydrocortisone cream
  • Severe= oral antifungal eg. terbinafine
418
Q

Referral to dermatology should be arranged in tinea pedis (athelete’s foot) when?

A
  • severe or topical antifungal unsuccessful in children
  • uncertain
  • Mx unsuccessful
  • immunocompromised
419
Q

CP of tinea pedis (athelete’s foot)?

A
  • Interdigital type= white or red, fissured, scaling skin or macerated areas between the toes. Lateral interdigital space between the 4th & 5th toes and then extends medially.
  • Moccasin or dry-type= more diffuse, chronic, causing scaling, erythema, and hyperkeratosis of the sole and lateral aspect of the foot. The dorsal surface unaffected.
  • Vesicobullous type= inflammatory variant with hard, tense, small (1–5 mm) vesicles, blisters, bullae, and pustules on an erythematous base, on the arches and soles of the feet.
420
Q

Advice on self care Mx for tinea pedis (athelete’s foot)?

A
  • well fitting non-occlusive footwear that keeps feet cool and dry
  • consider replacing old footwear that could be contaminated with fungal spores
  • wear different pair of shoes every 2-3days
  • cotton absorbent socks
  • avoid scratching
  • dry feet and between toes after washing
  • don’t share towels & wash frequently
  • CAN STILL GO SCHOOL
  • wear protective footwear eg. in swimming pools
421
Q

Advice on self-care Mx for tinea capitis (scalp ringworm)?

A
  • loose fitting clothes made of cotton or material to keep moisture away from skin
  • good hygiene: wash affected areas daily
  • avoid scratching
  • after washing dry thoroughly, esp skin folds
  • wash clothes and bed linen frequently
  • CAN STILL GO SCHOOL
422
Q

Urticaria?

A

Superficial swelling of the skin (epidermis and mucous membranes) that results in a red raised itchy rash.

‘hives’ or ‘nettle rash’

423
Q

What is a deeper form of urticaria?

A

Angio-oedema: deeper form of urticaria with swelling in the dermis and submucosal or subcutaneous tissue

424
Q

How is urticaria classfied?

A
  • Acute= symptoms last <6w
  • Chronic= symptoms persist for 6w+ on nearly daily basis
425
Q

How is chronic urticaria further classified?

A
  • chronic spontaneous (idiopathic) urticaria
  • autoimmune urticaria
  • chronic inducible urticaria: CINDU (chronic physical urticaria)
426
Q

Chronic spontaneous urticaria?

A

no identifiable external cause but may be aggravated by heat, stress, drugs, infections

427
Q

Autoimmune urticaria is characterised by the presence of what autoantibodies?

A

IgG autoantibodies to the high affinity receptor for IgE (Fc epsilon R1)

428
Q

Chronic inducible urticaria (CINDU)?

A

occurs in response to physical stimulus; can be classified according to its cause= aquagenic, cholinergic, solar, cold, heat, dermatographism, delayed pressure, vibratory, contact urticaria

429
Q

Ix for urticaria?

A

Clinical

Not required but may be done to identify trigger/exclude DD= LFTs, TFTs, ESR, CRP, FBC, allergy testing, urinalysis

430
Q

Mx for urticaria?

A
  • TUC
  • acute is likely to be self-limiting without treatment (if mild & avoidable trigger)
  • If symptoms require treatment….
    1) non sedating antihistaine daily for up to 6w eg. cetirizine
    2) severe= up to 7d of oral corticosteroids eg. presnisolone 40mg od up to 7d
431
Q

In pts with urticaria, if symptoms improve, what should you still consider?

A

if thought symptoms will be recurrent or persistent= antihistamine daily for 3-6m

if infrequent symptoms= antihistamine as required prophylactically

432
Q

What to do in the Mx of urticaria if response to treatment is inadequate?

A

Consider one of the following…

1) increase dose of 1st line antihistamine gradually up to 4x the licensed dose

2) an alternative non-sedating antihistamine

3) + topical antipuritic agent eg. calamine lotion

4) if itch interferring with sleep then + sedating antihistamine at night eg. chlorphenamine

5) refer to derm or immuno

433
Q

When should referral to derm or immuno be arranged in pt with urticaria?

A
  • vasculitic urticaria is suspected
  • pt has food or latex allergy or a form of CINDU that may be difficult to manage in primary care eg. solar or cold urticaria
434
Q

What is vasculitic urticaria?

A

inflam of blood vessels due to autoimmune reaction

suspect if urticaria is painful and persisitent

435
Q

CP of urticaria?

A
  • acute or chronic
  • may be episodic, hrs-days and recurring over months-yrs
  • central swelling of variable size, red or white in colour almost invariably surrounded by area of redness (flare
  • associated itchy and sometimes burning sensation
  • fleeting nature, skin returns to normal usually within 1-24hrs
436
Q

Erythema multiforme?

A

hypersensitivity reaction most commonly triggered by infections; minor and major.

437
Q

Features of erythema multiforme?

A
  • target lesions
  • initially seen on back of hands/feet before spreading to torso
  • upper limbs more commonly affected than lower
  • pruritis seen and mild
438
Q

Causes of erythema multiforme?

A
  • Virus: HSV (most common), Orf
  • idiopathic
  • bacteria: mycoplasma, strep
  • drugs
  • connective tissue disease eg. SLE
  • sarcoidosis
  • malignancy
439
Q

What drugs may cause erythema multiforme?

A

penicillin, NSAIDs, sulphonamides, carbamazepine, allopurinol, oral contraceptive pill, nevirapine

440
Q

Erythema multiforme major?

A

Most severe form of erythema multiforme, associated with mucosal invl.

441
Q

What is Orf?

A

Skin disease of sheep and goats caused by a parapox virus; cause of erythema multiforme

442
Q

Erythema condosum?

A

Inflamm of subcut fat

443
Q

CP of erythema nodosum?

A

tender, erythmatous, nodular lesions; usually skins but can occur elsewhere eg. forearms and thighs

444
Q

When does erythema nodosum typically resolve?

A

Within 6w;
lesions heal without scarring

445
Q

Causes of erythema nodosum?

A
  • infection: strep, TB, brucellosis
  • systemic disease: sarcoidosis, IBD, Behcet’s
  • malignancy/lymphoma
  • drugs
  • preganancy
446
Q

What drugs might cause erythema nodosum?

A

penicillins, sulphonamides, COCP

447
Q

Pityriasis rosea?

A

Self-limiting skin rash characterised by distinctive scaly erythmatous lesions and in some people, a hearld patch

448
Q

When may a herald patch appear in some pts with pityriasis rosea?

A

5-15d before the more generalised rash

449
Q

Cx of pityriasis rosea?

A
  • normally none
  • in 1st 15w of pregnancy: adverse outcomes eg. miscarriage and preamature delivery
450
Q

Diagnosis of pityriasis rosea?

A

Clinical

451
Q

CP of pityriasis rosea?

A
  • typically starts with herald patch before generalised eruption
  • discrete lesions, pink-red (salmon coloured) or fawn coloured, flat/slightly raised; circular/oval; 0.5-1cm; usually slightly scaly (confined to edge of lesion with central clearance)
452
Q

Distribution of lesions in pityriasis rosea?

A
  • distribution of lesions symmetircal; most on trunk (christmas tree pattern on upper back and V shape on upper chest); proximal limbs and few distal to mid-upper arm and mid-thigh
453
Q

Mx of pityriasis rosea?

A
  • Most no treatment needed, may worsen before resolves and new lesions up to 6w.
  • Itch= emollient or mild/moderately potent topical corticosteroid
  • Pregnant= discuss Mx urgently with secondary care
454
Q

How long for pityriasis rosea to settle without treatment?

A

2-3m, may take 5m to disappear.
Once disappeared may be hyper/hypopigmentation of affected skin for few months but no scarring

455
Q

What is a herald patch in pityriasis rosea?

A

larger than subsequent lesions, 2-5cm diameter but sometimes much larger; on trunk

456
Q

What is thought to play a role in pityriasis roasea?

A

HHV-7

457
Q

A minority of pts with pityriasis rosea may give a history of what?

A

Recent viral infection

458
Q

Forms of bite wounds?

A

abrasions, lacerations, puncture, crush or degloving

459
Q

Human bites are usually what?

A
  • occlusal injuries (from biting) or
  • clenched-fist injuries (when clenched fist hits person’s teeth cause small wounds over metacapophalangeal joints
460
Q

What do dog bites characteristically involve?

A

puncture wounds from canine teeth which anchor whilst other teeth bite, shear and tear tissues -> structural damage

461
Q

What do cat bites characteristically involve?

A

Cats= fine sharp teeth, weaker bite then dogs. But inflict deep punture wounds inoculated with salvia and capable of penetrating bone, joints and tendons

462
Q

Most common mammalian bite?

A

Dog

463
Q

Infective Cx of bites?

A

cellulitis, abscesses, tenosynovitis, septic arthritis, osteomyelitis, systemic spread (sepsis, meningitis).

Tetanus esp. if puncture wound or contains dirt/foreign body. Rare.

Rabies VERY rare in UK (none except bats are a risk), concern in Africa and Asia

464
Q

What should be assessed in pt with bite?

A
  • circumstances documented
  • risk of tetanus and blood-borne virus infection (for human bite wounds)
  • risk of rabies (animal bite)
465
Q

What does Mx of bite involve?

A
  • removal of foreign bodies
  • encourage wound to bleed
  • thorough irrigation with warm running water or saline
  • ?need for debridement or wound closure
  • analgesia
  • ?prophylactic Abx
  • Immediate advice from consultant in infectious disease if at risk of HIV or Hep B
  • ? tetanus and rabies prophylaxis
466
Q

When should prophylactic Abx be prescribed following a bite?

A

If skin broken and drawn blood; invl. high risk areas of skin; pt at risk of wound infections; penetrated vital tissues; deep wound; clinically infected.

467
Q

Intercainine distance of <3cm suggests what? (bite)

A

child bite; >3cm is adult bite

468
Q

Prophylactic Abx for bites if indicated?

A
  • co-amoxiclav 3d (allergic then metronidazole + doxy)
  • infected= co-amoxiclav 5d
469
Q

Assess risk of tetanus following a bite?

A
  • tetanus immunisation status
  • ?tetanus prone wound eg. contamination with soil, foreign body
470
Q

Assess risk of acquiring blood-borne viral infection eg. Hep B/C or HIV?

A
  • hep B vaccination status
  • already known to be +ve
  • blood tests
  • typically only if been exposed to the biters blood
471
Q

If pt is bitten, should puncture wounds be sutured?

A

No unless cosmesis at risk

472
Q

Typical organism from animal bites (cat and dog)?

A

polymicrobial but most common isolated is Pasteurella multocida

473
Q

Common organisms from human bites causing infection?

A

multimicrobial (aerobic and anaerobic)
- step spp.
- staph aureus
- Eikenella
- Fusobacterium
- Prevotella

474
Q

Bed bugs can cause what clinical problems?

A

itchy skin rashes, bites, allergic symptoms secondary to infestation with Cimex hemipteru

475
Q

Where do bedbugs thrive?

A

mattressess or fabrics; can be very hard to eradicate

476
Q

Mx for bedbugs?

A
  • topical hydrocortisone for itch
  • definitive= pest company & fumigation of house
  • numbers may be controlled by hot-washing bed linen and using mattress covers
477
Q

Lyme disease is an infection caused by group of bacteria called?

A

Borrelia burgforferi

478
Q

How is Lyme disease transmitted to humans?

A

following bite from infected tick

479
Q

RFs for Lyme disease?

A

exposure to woodland and fields; increased duration of tick attachment

480
Q

Cx of Lyme disease?

A

severe neuro symptoms; acrodermatitis chronica atrophans; lyme arthritis; persisting non-specific symptoms (sometimes called post-treatment Lyme disease syndrome)

481
Q

Erythema migrans rash only occurs in what?

A

Lyme disease

482
Q

Ix for Lyme disease?

A

Diagnostic = clinical if erythema migrans rash

1st= no rash- ELISA (enzyme-linked immunosorbent assay) for Lyme disease
-> if +ve or equivocal then perform immunoblot test for Lyme disease

483
Q
A
484
Q

When should pt be prescribed Abx in Lyme disease?

A
  • if have erythema migrans rash
  • If +ve ELISA, whilst waiting for immunoblot test results
485
Q

What if ?Lyme disease and ELISA negative but pt still has symptoms?

A

reviewed and if still suspected within 4w of onset then repeat ELISA 4-6 after 1st ELISA; if suspected and had symptoms 12w+ then immunoblot test

486
Q

In suspected Lyme disease, what if immunoblot test is positive? Negative with symptoms? Negative without symptoms?

A

+ve= oral Abx

-ve and symptoms= referral to specialist regardless of ELISA result

-ve and no symptoms= no treatment needed

487
Q

Mx for Lyme disease?

A

Doxycycline 100mg twice daily (or 200mg OD) for 21 days

If pregnant= amoxicillin 1000mg 3x daily for 21d

488
Q

Early features of Lyme disease? (within 30 days)

A
  • erythema migrans (80%)
  • systemic features= headache, lethargy, fever, arthralgia
489
Q

Later features of Lyme disease? (after 30 days)

A
  • CV: heart block, peri/myocarditis
  • neuro: facial nerve palsy, radicular pain, meningitis
490
Q

Jarisch-Herxheimer reaction?

A

Sometimes seen after initiating treatment for Lyme disease. Fever, rash, tachycardia after 1st dose Abx (more commonly seen in syphilis). 1-12hrs+ after, lasts hrs to 1-2days. Self-limiting. Continue Abx if no allergic reaction.

491
Q

What if a pt has a recent history of a tick bite but is otherwise well?

A

Abx prophylaxis not recommended but seek urgent advice is develop symptoms

492
Q

What to do it pt presents with a tick attached to skin?

A

Remove asap: fine-tipped tweezers, grasp as close to skin and pulling upwards firmly, wash area and keep an eye on bite area for several weeks.

493
Q

Burn?

A

injury caused by exposure to thermal (heat), electrical, chemical or radiation energy

494
Q

Scald?

A

burn caused by contact with a hot liquid or steam

495
Q

What do complex burns include?

A
  • all electrical & chemical burns
  • any thermal burn affecting critical area
  • any thermal burn covering >15% of total body SA in adults or >10% in children (>5% in <1yrs)
496
Q

Burns to where are classed as critical areas?

A

face, hands, feet, perineum, genitalia; crossing joints; circumferential burns

497
Q

What do non-complex burns include?

A
  • any partial-thickness burn covering up to 15% TNBA in adults, up to 10% in children and up to 5% <1yrs that does not affect critical area
  • a deep partial-thickness burn covering up to 1% of body
498
Q

Mx burns?

A
  • immediate first aid: remove person from source of burn and assess A, B, C, D, E
  • Non-complex= primary care Mx
499
Q

How is the severity of a burn determined?

A

location, extent, depth, age, presence of other injuries or cormorbidities

500
Q

Most burns are what?

A

Non-complex and can be managed primary care

501
Q

When should immediate admission to A&E be arranged for burns?

A
  • all complex burn injuries
  • all full-thickness burns
  • deep dermal burns affecting >5% TBSA adults
  • all deep dermal burns in children
  • circumferential deep dermal burns
  • any-high pressure steam injury
  • any burn ?non-accidential injury
  • affects critical area
  • associated with ?inhalation injury or sepsis
  • signif cormorbities that affect healing or risk of Cx
  • other signif injuries
502
Q

Primary care Mx of burn?

A
  • wound Mx
  • analgesia
  • skin care advice= cool bath/shower, cold compresses, massage area with emollient (emulsifying ointment) to relieve itching and dryness, protect skin from direct sunlight
  • ?tetanus prophylaxis
  • ?infected
503
Q

Cx of burns?

A

infection, hypotension, inhalation injury (black carbon in sputum, hoarse voice, stridor, wheeze)

504
Q

How to assess burn severity?

A
  • location & size
  • extent (TBSA- don’t include erythema)
  • depth: may increase with time so reassess after 24-72hrs essential
505
Q

How to assess extent of burns?

A

Lund and Browder method: add burned areas up on chart to calculate TBSA affected
or
Wallace’s rule of nines

506
Q

The depth of burn injuries can be classified into what groups?

A

Superficial partial thickness burns (1st & 2nd degree):
- superficial epidermal burns (s. 1st degree)
- superficial dermal burns (s. partial-thickness burns)
- deep dermal burns (d. partial thickness)

and

Full thickness burns (3rd degree) and Severe full thickness burns (4th degree)

507
Q

Layers of skin affected and CP of a superficial epidermal burn (1st degree) eg. suburn?

A
  • epidermis but dermis intact
  • red and painful, not blistered; cap refill blanches then rapidly refills
508
Q

Layers of skin affected and CP of superficial dermal (partial thickness) burn?

A
  • epidermis and upper layer dermis affected
  • red or pale pink with painful blistering; cap refill blanches but regains colour slowly
509
Q

Layers of skin affected and CP of deep dermal (partial thickness) burn?

A
  • epidermis, upper and deeper layers of dermis affected but not underlying subcut tissues
  • dry, blotchy or mottled, red and painful, may be blisters; cap refill does not blanch
510
Q

Layers of skin affected and CP of full thickness burns (3rd and 4th degree)?

A
  • all layers incl subcutanous tissues; if severe it extends into muscle and bone
  • white, brown or black (charred), no blisters; dry, leathery or waxy; painless; cap refill doesn’t blanch
511
Q

How to check cap refill in burns?

A

pressing with sterile cotton bud eg. bacteriology swab

512
Q

Mx for thermal burns?

A
  • remove non-adherent clothing and jewellery, do NOT attempt to remove tar stuck to skin
  • within 20mins= irrigate burn with cool running water 15-30mins (do not use ice or v cold water), or use wet towels if no water
  • immediately after cooling cover burn with cling film: layer over not wrap to avoid constriction; or use clean cotton sheet
  • elevate area to reduce risk of oedema
  • analgesia

-?severity, is admission needed

  • don’t apply topical creams at this stage as will hinder wound assessment
513
Q

Mx for electrical burns?

A
  • don’t approach if connected to high voltage source (1000v +)
  • If low-voltage source (domestic 220-240v) then switch off power supply and remove person using non-conductive material eg. wooden chair
  • immediate admission to A&E
  • assess severity
514
Q

Mx for chemical burns?

A
  • remove affected clothing
  • irrigate the burn with copious amounts of water for 1hr
  • don’t attempt to neutralise chemicals
  • immediate A&E admission
  • severity
515
Q

Why don’t you use ice or very cold water in burn Mx?

A

may cause vasoconstriction and deepen the wound

516
Q

Wound care for burns in primary care?

A
  • cleaned and dressed with non-adherent dressing eg. paraffin gauze + secondary non-fibrous absorbent dressing eg. dressing pad
  • after 48hrs= dressing change and reassess
  • every 3-5days until wound healed= dressing changes
  • XS exudate may be normal finding in 1st 72hrs following burn
  • leave blisters in tact
517
Q

Mx of more severe burns?

A
  • ?early intubation
  • IV fluids (v=TBSA of burn x weight in kg x 4): half fluid is administered in 1st 8hrs

-urinary catheter, analgesia

  • transfer to burns unit if complex
  • circumferential burns or severe torso burns= ?escharotomy to divide burnt tissue (improve ventillation or relieve compartment syndrome/oedema)
  • complex= ?skin grafting
518
Q

Types of birth marks (congenital naevi)?

A
  • salmon patches/stork marks
  • strawberry marks/hemangiomas
  • port wine stains
  • cafe-au-lait spots
  • Mongolian blue spots
  • congenital moles or congenital melanocytic naevi
519
Q
A
520
Q

Cafe-au-lait spots?

A

light or dark brown patches anywhere on body; common, many have 1/2; diff sizes and shapes; if >6 then may be sign of neurofibromatosis type 1

521
Q
A
522
Q

Mongolian blue spots?

A

blue-grey like bruise; often lower back, bottom, arm or leg; there from birth; common on darker skin; don’t need Mx; go away by 4yrs; not sign of condition but should be recorded on medical record

523
Q
A
524
Q

Congenital moles or congenital melanocytic naevi?

A

brown or black moles due to overgrowth of pigment cells in skin; darker on darker skin; can become darker raised and hairy espec during puberty; don’t need Mx unless risk of skin ca

525
Q
A
526
Q

Mx of birth marks?

A

most don’t need Mx; some drugs can reduce blood slower so make it lighter and slow down growth; laser therapy (best between 6m-1yr); surgery

527
Q

Red flags of birth marks?

A

close to eye, nose or mouth; gotten bigger, darker or lumpier; sore or painful; 6+ cafe au lait; large congenital mole

528
Q

Lichen planus?

A

skin disorder of unknown aetiology (?immune mediated)

529
Q

CP of lichen planus?

A
  • itchy papular rash
  • common on palms, soles, genitalia and flexor surfaces of arms
  • polygonal in shape
  • ‘white lines’ pattern on surface (Whickham’s straie)
  • Koebner phenomenon may be seen
  • oral invl in 50% pts: white-lace pattern on buccal mucosa
  • nails: thinning, longitudinal ridging
530
Q

Koebner phenomenon in lichen planus?

A

new skin lesions appearing at site of trauma

531
Q

Causes of Lichenoid drug eruptions (lichen planus)?

A
  • gold
  • quinine
  • thiazides
532
Q

Mx for lichen planus?

A
  • potent topical steroids
  • oral: benzydamine mouthwash/spray
  • extensive: oral steroids
533
Q

Rosacea (acne rosacea)?

A

chronic inflam skin condition affecting convexities of the centrofacial region (cheeks, chin, nose, central part of forehead)

534
Q

Diagnosis of rosacea?

A

Clinical using phenotype approach: at least 1 diagnostic features or 2 major clinical features

535
Q

Features of rosacea?

A
  • Diagnostic= phymatous changes, persistent erythema
  • Major= flushing/transient erythema, papules and pustles, telangiectasia, eye symptoms (ocular rosacea)
  • Minor= skin burning and/or stinging sensation, skin dryness, oedema
536
Q

Ocular rosacea?

A

eye discomfort, irritation, tearing, foreign body sensation, dryness, itching, photophobia or blurred vision

might present as: lid margin telangiectasia, blepharitis or acute lid infection, conjunctivitis, keratitis, or anterior uveitis

may be without skin disease

537
Q

Rosacea over time?

A

May progress in severity; repeated remissions and exacerbations

538
Q

Cause of rosacea is unknown but what are some RFs?

A
  • age
  • photosensitive skin types
  • UV raditation exposure
  • smoking
  • alcohol
  • spicy food and hot drinks
  • hot or cold temps
  • stress and exercise
  • colonisation with Demodex folliculorum mites
539
Q

Mx of rosacea?

A
  • self Mx advice
  • referral to skin camouflage service if appropriate
  • topical and/or oral drug treatments
540
Q

Topical and/or oral drug treatments for rosacea?

A
  • persistent= topical brimonidine
  • mild-moderate= topical ivermectin
  • moderate-severe= + oral doxycycline
541
Q

When to refer to derm for rosacea?

A
  • not responded to primary care Mx
  • severe telangiectasia not responded to self-Mx advice
  • uncertain
542
Q

When to refer to plastic surgeon for rosacea?

A
  • prominent non-inflamed phymatous disease
543
Q

When to refer to opthalmology for rosacea?

A
  • serious eye Cx suspected eg. keratitis or anterior uveitis
  • other Cx not responded to self-care Mx eg. lid hygiene measures and artifical tears/ocular lubricants
544
Q

Self-care Mx advice for rosacea?

A

trigger avoidance, sun protection, skin care measures

545
Q

What may exacerbate symptoms in rosacea?

A

sunlight

546
Q

In rosacea, what might be appropriate for pts with prominent telangiectasia?

A

Laser therapy

547
Q

Seborrhoeic dermatitis?

A

Common inflam skin condition occuring in areas rich in sebaceous glands eg. scalp, nasolabial folds, eyebrows and chest

548
Q

Where does seborrhoeic dermatitis affect infants the most?

A

scalp: ‘cradle cap’

549
Q

How does seborrhoeic dermatitis present?

A

Erythematous patches with greasy-looking yellowish scales

550
Q

Cause of seborrhoeic dermatitis?

A

Not understood; Malassezia furfur yeasts and host immune response associtated

551
Q

Infantile seborrhoeic dermatitis?

A

usually self-limiting, most common in 1st 3m of life; normally resolves at around 8-12m

552
Q

Is seborrhoeic dermatitis chronic or acute in adults?

A

Chronic: fluctuates over time, responds well to Mx but may relapse and remit over yrs

553
Q

Ix for seborrhoeic dermatitis?

A

1st/diagnosis= clinical
2nd= bloods, skin scrapings or skin biopsy if suspect underlying cause/diagnosis eg. HIV, tinea, lupus

554
Q

Mx of seborrhoeic dermatitis in infants?

A
  • massage topical emollient onto the scalp to loosen scales and remove them with an infant brush before shampoo
  • bathe infant every day using barrier emollient as soap substitute= zince & caster oil ointment BP
  • consider topical imidazole cream= clotrimazole 1% cream 2-3x daily up to 4w
  • No response or severe= short course low potency topical corticosteroids (1% hydrocortisone OD up to 2w)
555
Q

Mx of seborrhoeic dermatitis of scalp and beard?

A
  • Ketoconazole 2% shampoo (twice week up to 4w then once every 1-2w maintenance)
  • Consider short course topical corticosteroid scalp application (betamethasone valerate 0.1% scalp application once/twice a day): reduce inflam and itching
  • don’t use potent steroid scalp application on beard, use low potency hydrocortisone 1%
556
Q

Mx of seborrheoic dermatitis of face and body in adults?

A
  • ketoconazole 2% cream once/twice a day up to 4w (adults) + antifungal shampoo eg. ketoconazole 2% to use as body wash if appropraite

-Consider short course low-medium potency topical corticosteroid cream for flares eg. hydrocortisone 1% or betamethasone valerate 0.1%

557
Q

What Mx advise could you give pt who feels like seborrhoeic dermatitis is under control?

A

ketoconazole 2% cream can be used less frequently to prevent recurrence eg. twice/once week, once every other week

558
Q

Mx of seborrheoic dermatitis of face and body in children and adolescents?

A
  • same as adults but clotrimazole 2-3x a day up to 4w not ketoconazole
  • can still use ketoconazole shampoo as body wash if >12yrs
559
Q

When should pts with seborrhoeic dermatitis be referred to derm?

A
  • uncertain
  • failure to respond to Mx
  • severe
  • eyelid invl. (eyelid hygiene measures unsuccessful
560
Q

CP of seborrhoeic dermatitis in general?

A
  • well defined erythematous plaques with superficial greasy looking white or yellow scales
  • darker skin= may be hypopigmented scaly patches and erythema harder to see
561
Q

CP of seborrhoeic dermatitis in certain areas?

A
  • face: nasolabial folds, glabellar area, eyebrows & behind ear= yellowish scaling between eyelashes and blepharitis; crusting, oozing, fissures behind ears that may spread to external ear canal
  • scalp: mild desquamation (dandruff) or honey-coloured crusts that adhere to skin and hair; pruritus
  • upper chest and back: petaloid type (petal shaped lesions on upper chest) and pityriasiform type (widespread oval scaly macules and patches distributed similar to extensive pityriasis rosea
562
Q

CP of seborrhoeic dermatitis in infants in specific areas?

A

Mainly extensive coverage of scalp (cradle cap).

scalp= non-inflam eruption of thick white, red or yellow greasy scales mainly affected vertex and frontal areas

  • face= erythematous, flaky, salmon-coloured plaques on forehead, eyebrows, eyelids, nasolabial folds or retro-auricular

body folds= moist, shiny, non-scaly aspects, tend to coalesce on neck, axillae or inguinal area

trunk= sharply limited plaques of erythema and scaling over lower abdo

may be on nappy area

563
Q

What else might be present in infants with seborrhoeic dermatitis?

A

generalised rash, like atopic eczema, with widespread erythematous scaly lesions but with little itch

564
Q

What does cradle cap (seborrhoeic dermatitis) look in children of colour?

A

erythema, flaking and hypopigmented affected areas and overlying atopic dermatitis which accentuates hypopigmentation

565
Q

What might an explosive, often generalised, onset of seborrhoeic dermatitis be a marker for?

A

HIV

566
Q
A
567
Q

What may develop in pt with seborrhoeic dermatitis?

A

otitis externa and blepharitis

568
Q

Fungal nail infection (oncychomycosis) can involve where?

A

Any part of nail or entire nail unit, incl. nail plate, nail bed and root of nail.
Nail unit may discolour, nail plate may distort and the nail bed & adjacent tissue may thicken.
Toenails 7x likely > fingernails

569
Q

What causes fungal nail infection?

A

Dermatophyte (90%) (eg. Trichophyton rubrum)
Non-dermatophyte moulds
Yeasts eg. Candida species

570
Q
A
571
Q

RFs for fungal nail?

A

age; athletes; psoriasis; DM; PAD; immunocompromised; repeated nail trauma; occlusive footwear; concomittant fungal skin infection

572
Q

CP of fungal nail?

A
  • abnormal and discoloured
  • white or yellow streaks along one side of nail
  • thickening of nail
  • white or yellow spots
  • complete destruction of nail
  • associated paronychia- ?Candida
573
Q

Ix for fungal nail?

A
  • Clinical
  • Confirm diagnosis if pt wants treatment= nail clippings and/or scrapings for fungal microscopy and culture
574
Q

Mx for fungal nail infection?

A

1st= self-care strategies

2nd= antifungal treatment

3rd= re-sample nail clippings; topical + oral antigungal

575
Q

Antifungal treatment for fungal nail?

A
  • limited involvement (≤50% nail affected, ≤ 2 nails affected, more superficial white onychomycosis): topical treatment with amorolfine 5% nail lacquer; 6 months for fingernails and 9 - 12 months for toenails

-more extensive involvement due to a dermatophyte infection= oral terbinafine is currently recommended first-line; 6 weeks - 3 months therapy is needed for fingernail infections whilst toenails should be treated for 3 - 6 months

-more extensive involvement due to a Candida infection= oral itraconazole is recommended first-line; ‘pulsed’ weekly therapy is recommended

576
Q

When to refer pt with fungal nail to podiatrist?

A
  • discomfort walking due to thickened nails
  • nail trauma due to footwear
  • deformed nails trumatising adjacent toes
577
Q

When to refer pt with fungal nail to derm?

A
  • uncertain
  • oral antifungal considered for a child
  • Mx unsuccessfuk
  • co-existant nail disease eg. psorriasis or lichen planus
  • immunocompromised
578
Q

Dose of oral terbinafine for dermatophyte fungal nail infection?

A

250mg OD 6w for fingernails and 12-24w for toenail; improvement after 2m for finger or 3m toenail

579
Q

When to review pt for subsequent nail growth if have fungal nail?

A

3-6m after starting treatment

580
Q

When should antifungal treatment for fungal nail not be advised and advised?

A
  • not needed if not troubled by appearance of nail or infection is asymtomatic
  • advised= walking uncomfortable, pyschological distress from appearance, nail infection as source of associated fungal skin infection
581
Q

Self-care advise for fungal nail?

A

nails short and filed; avoid sharing clippers; well-fitting non-occlusive shoes; cotton absorbent socks; good foot hygiene

582
Q

Differential diagnosis for fungal nail infection?

A

psoriasis
repeated trauma
lichen planus
yellow nail syndrome

583
Q

What conditions can cause nail pitting?

A

psoriasis
alopecia acreata

584
Q

What conditions can cause blue nails?

A

Wilson’s

585
Q

What conditions can cause leuconychia?

A

hypoalbuminaemia
less common= fungal disease, lymphome

586
Q

What conditions can cause half and half nails?

A

eg. distal portion brown and proximal portion pale= chronic renal failure

587
Q

What conditions can cause yellow nail syndrome?

A

congenital lymphoedema, pleural effusions, bronchiectasis, chronic sinus infections

588
Q

What is onycholysis?

A

Separation of the nail plate from the nail bed

589
Q

Causes of onycholysis?

A

idiopathic
trauma eg. XS manicure
infection eg. fungal
psoriasis, dermatitis
impaired peripheral circulation eg. Raynaud’s
hyper and hypothyroidism

590
Q

What % of pt with psoriatic arthropathy have nail changes?

A

80-90%

591
Q

Nail changes that may be seen in psoriasis?

A

pitting, oncholysis, subungal hyperkeratosis, loss of nail

592
Q

Causes of pruitus?

A

Liver disease; iron def anaemia; polycythaemia; CKD; lymphoma; hyper & hypothyroidism; DM; pregnancy; ‘senile’ pruritis; urticaria; skin disorders: eczema, scabies, psoriasis, pityriasis rosea

593
Q

Characteristics of lymphoma?

A

night sweats, lymphadenopathy, splenomegaly, hepatomegaly, fatigue, pruritis

594
Q

Keloid scars?

A

tumour-like lesions that arise from connective tissue of a scar and extend beyond the dimensions of the original wound

595
Q

Predisposing factors to keloid scars?

A

ethnicity (darker skin more common); young adults; common sites= sternum, shoulder, neck, face, extensor surface of limbs, trunk

596
Q

What makes keloid scars less likely?

A

if incisions are made along relaxed skin tension lines

597
Q

Mx for keloid scars?

A
  • early: intra-lesional steroids eg. triamcinolone
  • excision is sometimes required but consideration as may create further keloid scarring
598
Q

Actinic keratoses (solar keratoses)? (AK)

A

common premalignant skin lesion that develops as consequence of chronic sun exposure

599
Q

Features of actinic keratoses?

A
  • small, crusty or scaly lesions
  • pink, brown, red or same colour as the skin
  • typically on sun-exposed ares eg. temples of head
  • multiple lesions may be present
600
Q

Prevention of further risk of actinic keratoses?

A

sun avoidance, sun cream

601
Q

Mx options for actinic keratoses?

A
  • fluorouracil cream= 2-3w course; can cause red and inflam so topical hydrocortisone may be given too
  • topical diclofenac= mild AK; moderate efficacy but much fewer S/Es
  • topical imiquimod
  • cyrotherapy
  • curettage and cautery
602
Q

Lipoma?

A

benign tumour of adipocytes; common; middle-aged adults commonly; 1 in 1000

603
Q

Pathophysiology of lipoma?

A

found in subcut tissue; rarely can be found in deeper adipose tissue; malignant transformation to liposarcoma very rare

604
Q

Features of lipoma?

A

smooth, mobile, painless

605
Q

Ix/diagnosis for lipoma?

A

Clinical

606
Q

Mx for lipoma?

A
  • observed
  • if uncertain or compressing surrounding structures it may be removed
607
Q

Liposarcoma features?

A

features suggestive of sarcomatous change:
- >5cm
- increasing size
- pain
- deep anatomical location

608
Q

Venous leg ulcers: due to what?

A

venous HTN, 2 to chronic venous insufficiency, calf pump dysfunction, neuromuscular disorders

Form due to capillary fibrin cuff or leucocyte sequestration

609
Q

Features of venous leg ulcers?

A

oedema, brown pigmentation, lipodermatosclerosis, eczema

above the ankle, painless

610
Q

Deep venous insufficiency is related to what? Superficial venous insufficiency?

A

deep= previous DVT
superficial= varicose veins

611
Q

Venous leg ulcers: doppler USS vs duplex USS?

A
  • doppler= presence of reflux, speed and direction of blood flow
  • duplex= anatomy/flow of vein
612
Q

Mx for lower leg ulcers?

A

4 layer compression banding after exclusion of arterial disease or surgery.
Fail to heal after 12w or >10cm2 then skin grafting may be needed

613
Q

Marjolin’s ulcer?

A

SCC; occuring at sites of chronic inflamm eg. burns, osteomyelitis after 10-20yrs; mainly on lower limb

614
Q

Arterial ulcers?

A
  • mainly on toes and heel
  • ‘deep, punched-out’ appearance
  • painful
  • may be areas of gangrene
  • cold with no palpable pulses
  • low ABPI
615
Q

Neuropathic ulcers?

A
  • plantar surface of metatarsal head and plantar surface of hallux
  • due to pressure
  • Mx: cushioned shoes to reduce callous formation
616
Q

Plantar neuropathic ulcer is a condition that most commonly leads to…

A

amputation in diabetic patients

617
Q

Pyoderma gangrenosum?

A
  • erythematous nodules or pustules which ulcerate
  • can occur at stoma sites
  • associated with IBD/RA
618
Q

Atrophic vaginitis occurs often in who?

A

Women who are post-menopausal

619
Q

Features of atrophic vaginitis?

A

vaginal dryness, dyspareunia, occassional spotting

on exam: pale and dry

620
Q

Mx for atrophic vaginitis?

A

vaginal lubricants and moisturisers
if don’t help then topical oestrogen cream

621
Q

Pruritus vulvae?

A

itching of the vulva

622
Q

Underlying causes of pruritus vulvae?

A
  • derm conditions
  • infections and infestations
  • neoplastic conditions
  • hormonal changes (atrophic in peri and post menopausal)
  • DM (increased risk of candidal infection)
  • GI disease and urinary incontinence
  • neuro conditions
  • systemic causes (renal or hepatic impairment)
623
Q

Cx that may result from underlying cause of pruritus vulvae?

A

small risk of SSC for women with lichen sclerosus and lichen planus

vulval intraepithelial neoplasia may progress to vulval ca

624
Q

Cx from chronic itch in pruritus vulvae?

A
  • psychological
  • sexual dysfunction
  • sleep disturbance
  • severe and irreversible architectural damage due to deep scratching and gouging
  • lichen simplex
625
Q

What is common in excoriated skin eg. in pruritus vulvae?

A

secondary bacterial infection

626
Q

Mx for pruritus vulvae?

A
  • Advice
  • TUC
  • symptomatic Mx with emollient
  • sleep affected= ?antihistamine (hydroxyzine) or tricyclic antidepressant (amitriptyline)
  • ?short trial (1-2w) low potency topical corticosteroid
  • ? corticosteroid + antifungal or corticosteroid + antibacterial if co-existing infection is suspected
627
Q

When to refer to derm or gynae for pruritus vulvae?

A
  • persistant
  • uncertain
  • urgent 2ww if vulval carcinoma suspected (unexplained lump, ulcer, bleeding, area of abnormality eg. thick, warty or ulcered)
628
Q

Aim of Mx for pruritus vulvae?

A

symptomatic relief, reduce inflamm, restore skin barrier and prevent/treat secondary infection

629
Q

Advice for women with pruritus vulvae?

A
  • shower rather than bath, with mild unscented emollient ointment and to clean vulval area OD; once clean gently dab dry; cooling the area may help
  • avoid: washing with water only or with soap; contact with shampoo, bubble bath, vaginal washes or wet wipes; tight fitting or synthetic underwear; fabric conditioner or fragranced washing powder; spermicidally lubricated condoms; scratching; having long nails; coloured or scented toilet paper
630
Q

Features of pruritus vulvae?

A
  • triggers= sexual activity, menses, exercise, heat, moisture, friction, XS washing; creams, perfumes, soaps
  • skin colour, texture, fissuring, excoriation, erosions, ulcerations, lichenification, lesions, tumours, atrophy, scarring, discharge.
  • look at anogenital region, vaginal mucosa with speculum and then extragenital sites eg. elbows, knees, nails, skin, oral mucosa
631
Q

Ix for pruritus vulvae?

A
  • clinical
  • high vaginal swab if discharge or signs of infection
  • Bloods if generalised pruritus
632
Q

Bloods for generalised pruritus?

A

FBC, ferritin, TFTs, renal function tests, LFTs, blood glucose, HbA1c

633
Q

Alopecia areata?

A

Autoimmune condition causing localised well demarcated patches of hair loss; may be small broken exclamation mark hairs at edge of hair loss

634
Q

Alopecia areata Mx?

A

Will regrow in 50% pts by 1 yr and 80-90% eventually so just explanation.
- other: topical corticosteroids, topical minoxidil, phototherapy, dithranol, contact immunotherapy, wigs

635
Q

Ix and Mx for bullous pemphigoid?

A
  • refer to derm for skin biopsy and confirmation= immunofluorescence shows IgG and C3 at dermoepidermal junction
  • oral corticosteroids
635
Q
A
636
Q
A
637
Q
A
638
Q
A
639
Q
A
640
Q

Cherry haemangioma? (Campbell de Morgan spots)

A

benign skin lesions containing abnorm proliferation of capillaries; advancing age

erythematous, papular lesions, 1-3mm, non-blanching, not found on mucous membranes

benign so no Mx needed

641
Q

Dermatitis herpetiformis?

A

autoimmune blistering skin disorder associated with coeliac disease; caused by deposition of IgA in dermis

itchy, vesicular skin lesions on extensor surfaces eg. elbows, knees, buttocks

642
Q

Ix and Mx for dermatitis herpetiformis?

A
  • skin biopsy= direct immunofluorescence shows deposition of IgA in granular pattern in upper dermis
  • gluten-free diet; dapsone
643
Q

Dermatofibroma? (histiocytomas)

A

common benign fibrous skin lesions caused by abnormal growth of dermal dendritic histiocyte cells, often following precipitating injury

common on arms and legs

644
Q

Features of dermatofibroma? (histiocytomas)

A

solitary firm papule or nodule, typically on a limb; 5-10mm; overlying skin dimples on pinching the lesion

645
Q

Erysipelas?

A

localised skin infection caused by strep pyogenes; more superficial limited version of cellulitis

646
Q

Mx for erysipelas?

A

flucloxacillin

647
Q

Erythrasma?

A

asymptomatic flat slightly scaly, pink or brown rash usually in groin or axillae; caused by overgrowth of diptheroid Conrynebacterium minutissimum

648
Q

Erythrasma Ix and Mx?

A

Exam with Wood’s light= coral-red fluorescence

Topical miconazole or antibacterial; extensive= oral erythromycin

649
Q

Hereditary haemorrhagic telangiectasis? (Osler-Weber-Rendu syndrome)

A

Autosomal dominant but 20% have no FHx

Multiple telangiectasia over skin and mucous membranes.

650
Q

Diagnostic criteria for hereditary haemorrhagic telangiectasia? If have 2 then possible, if 3+ then definite diagnosis.

A
  • Epistaxis: recurrent
  • Telangiectases: multiple sites (lips, oral cavity, fingers, nose)
  • Visceral lesions: GI Telang (with/without bleeding); pulmonary ateriovenous malformations (AVM); hepatic AVM; cerebral AVM; spinal AVM
  • FHx: 1st degree relative
651
Q

Hidradenitis suppurativa?

A

chronic painful inflam skin disorder; develop inflam nodules, pustules, sinus tracts and scars (rope-like scarring) in intertriginous areas (eg. axilla); suspect in pubertal/post-pubertal pts who have diagnosis of recurrent furuncles or boils in intertriginous areas.

Mx: hygiene, loose clothes. Acute: steroids (intralesional or oral); chronic: topical clindamycin or oral; may need excised surgically

652
Q

Hirsutism vs hypertrichosis?

A

Hirsutism= androgen-dependent hair growth in women

Hypertrichosis= androgen-independent hair growth

653
Q

Causes of hirsutism?

A

PCOS; cushings; congenital andrenal hyperplasia; androgen therapy; obesity (due to insulin resistance); phenytoin and corticosteroids

654
Q

Assessment of hirsutism?

A

Ferriman-Gallwey scoring system (9 body areas assigned score 0-4; >15 is moderate/severe hirsutism

655
Q

Mx of hirsutism?

A

weight loss; waxing/bleaching; COCP; topical eflornithine

656
Q

Causes of hypertrichosis?

A
  • drugs: minoxidil, ciclosporin, diazoxide
  • congenital hypertrichosis languninosa/terminalis
  • porphyria cutanea tarda
  • anorexia nervosa
657
Q

Mx for hyperhidrosis (XS production of sweat)?

A

1st: topical aluminium chloride (may cause skin irritation)

658
Q

Keratoacanthoma?

A

Benign epithelial tumour; common in older pts.

Look like volcano or crater; initally smooth dome-shaped papule then rapidly grows to become crater centrally filled with keratin

659
Q

Mx for keratoacanthoma?

A

spontaneous regression in 3m common, scars; urgently excise however as hard to exclude SCC

660
Q

Lentigo maligna?

A

Type of melanoma in-situ; progresses slowly but may become invasive causing lentigo maligna melanoma

661
Q

Leukoplakia?

A

premalignant condition

white, hard spots on mucous membranes of mouth, common in smokers

diagnosis of exclusion; if lesions can be rubbed off= candidiasis or lichen planus

biopsy to exclude SCC and regular follow up to exclude transition to SCC

662
Q

Livedo reticularis?

A

purplish, non-blanching, reticulated rash caused by obstruction of capillaries resulting in swollen venules

causes= idiopathic (common); SLE; antiphospholipid syndrome

663
Q

Milia?

A

Small benign, keratin-filled cysts that typically appear around face; common in newborns

664
Q

Itchy red papules typically confused with eczema or psoriasis but lesions are different colours in contrast to E or P where there is greater homogenicity?

A

Mycosis fungoides: rare form of T-cell lymphoma that affects skin

665
Q

Common cause of allergic contact dermatitis?

A

Nickel eg. watches

666
Q

Polymorphic eruption of pregnancy?

A

pruritic condition associated with the last trimester; often first appear in abdo striae; periumbilical area often spared

Mx: emollients, mild potency topical steroids or oral steroids depending on severity

667
Q

Porphyria cutanea tarda?

A

most common heptic porphyria, due to inherited defect in uroporphyrinogen decarboxylase or hepatocyte damage eg. alcohol, hep C, oestrogen

668
Q

Porphyria cutanea tarda CP?

A

photosensitive rash with blistering and skin fragility of face and dorsal aspects of hands; hypertichosis; hyperpigmentation

669
Q

Porphyria cutanea tarda Ix?

A
  • Urine= elevated uroporphyrinogen and pink fluorescence of urine under Wood’s lamp
  • Serum iron ferritin level used to guide therapy
670
Q

Porphyria cutanea tarda Mx?

A
  • chloroquine
  • if ferritin >600ng/ml then venesection
671
Q

Purpura?

A

Bleeding into skin from small blood vessels that produces non-blanching rash; small petechiae 1-2mm may be seen.
Children with new purpuric rash= admit immediately ?meningococcal septicaemia or ALL

672
Q

Causes of purpura in children?

A
  • meningococcal septicaemia
  • ALL
  • congenital bleeding disorders
  • immune thrombocytopenic purpura
  • Henoch-Schonlein purpura
  • non-accidental injury
673
Q

Causes of purpura in adults?

A
  • immune thrombocytopenic purpura
  • bone marrow failure (2 to leukaemia or bone mets)
  • senile purpura
  • drugs
  • Vit B12, C and folate def
674
Q

What drugs may cause purpura?

A

quinine, antiepileptics, antithrombotics

675
Q

Raised superior vena cava pressure eg. secondary to bad cough may cause what?

A

Petechiae in upper body; would not cause purpura

676
Q

Purpura vs petechiae?

A

Both types of bleeding into skin that can be caused by broken blood vessels.

Purpura: form areas >4mm
Petechiae: tiny red dots <4mm

677
Q

Pyogenic granuloma?

A

common benign skin lesion; name not helpful, more like ‘eruptive haemangioma’/
Ititally small red/brown spot; progresses rapid over days-w forming red/brown lesions often spherical in shape; may bleed or ulcerate.
Caused by trauma or pregnancy.
- Mx if pregnant: resolve spontaneously post-partum
- Others: persist so curettage and cauterisation; cryotherapyor excision

678
Q

Adverse effects of retinoids? (used in severe acne eg. oral isotretinoin)

A
  • teratogenicity
  • dry skin, eyes and lips/mouth (common with isotretinoin)
  • low mood
  • raised triglycerides
  • hair thinning
  • nose bleeds
  • intracranial HTN (isotretinoin should not be combined with tetracyclines)
  • photosensitivity
679
Q

Differential diagnosis of shin lesions?

A
  • erythema nodosum
  • pretibial myxoedema
  • pyoderma gangrenosum
  • necrobiosis lipoidica diabeticorum
680
Q

Pretibial myxoedema?

A

symmetrical erythematous lesions seen in Graves’

Shiny, orange peel skin

681
Q

Necrobiosis lipoidica diabeticorum?

A

Shiny painless areas of yellow/red skin typically on shin of diabetics; often assocaited with telangiectasis

682
Q

Atopic eruption of pregnancy?

A

Most common skin disorder found in pregnancy; eczematous itchy red rash; no Mx needed

683
Q

Pemphigoid gestationis?

A

Pruritic blistering lesions usually in 2nd or 3rd trimester of pregnancy; peri-umbilical region then spreads to trunk, back, buttocks, arms. Require oral corticosteroids.

684
Q

Skin disorders associated with pregnancy?

A

Atopic eruption of pregnancy (commonest)
Polymorphic eruption of P
Pemphigoid gestationis

685
Q

Spider naevi? (spider angiomas)

A

Central red papule with surrounding capillaries; blanch upon pressure; found on upper part of body.
10-15% people will have 1+, common in childhood; liver disease; pregnancy and COCP.

686
Q

How to differentiate spider naevi from telangiectasia?

A

Press on them and watch them fill. Spider naevi fill from centre, telangiectasia from edge.

687
Q

Stevens-Johnson syndrome?

A

Severe systemic reaction affecting skin and mucosa almost always caused by drug reaction.

688
Q

Causes of Stevens-Johnson syndrome?

A
  • penicillin
  • sulphonamides eg. sulfasalazine
  • lamotrigine, carbamazepine, phenytoin
  • allopurinol
  • NSAIDs
  • oral contraceptive pill
689
Q

CP of Stevens-Johnson syndrome?

A
  • maculopapular rash with target lesions being characterisitic
  • may develop into vesicles or billae
  • Nikolsky sign +ve in erythematous areas
  • mucosal involvement
  • systemic: fever, arthralgia
690
Q

Nikolsky sign in Stevens-Johnson syndrome?

A

Positive in erythematous areas- blisters and erosions appear when skin is rubbed gently

691
Q

Mx for Stevens-Johnson syndrome?

A

Hospital admission for supportive treatment

692
Q

Vacilitides?

A
  • Large vessel= temporal arteritis; Takayasu’s arteritis
  • Medium vessel= polyarteritis nodosa; Kawasaki
  • Small vessel= ANCA-associated (Wegener’s granulomatosis; eosinophilic granulomatosis with polyangiitis; microscopic polyangiitis) or immune complex small-vessel vasculitis (Henoch-Schonlein purpura; Goodpasture’s; cryoglobulinaemic vasculitis; hypocomplementemic urticarial vasculitis)
693
Q

Features of vitiligo?

A

well-demarcated patches of pigmented skin; peripheries most affected; trauma may precipitate new lesions (Koebner phenomenon)

694
Q
A
695
Q

Mx of vitiligo?

A

Sunblock for affected areas; camouflage makeup; topical corticosteroids may reverse changes if applied early

696
Q

Where is Koebner phenomenon seen in?

A
  • psoriasis
  • vitiligo
  • warts
  • lichen planus
  • lichen sclerosus
  • molluscum contagiosum
697
Q

Features of zinc def?

A
  • acrodermatitis: red crusted lesions; acral, peri-orficial, perianal
  • alopecia
  • short
  • hypogonadism
  • hepatosplenomegaly
  • geophagia (ingesting clay/soil)
  • cognitive impairment
698
Q

What is acrodermatitis enteropathica?

A

Recessively inherited partial defect in intestinal zinc absorption.

699
Q

Shingles (herpes zoster infection)?

A

acute, unilateral, painful blistering rash caused by reactivation of varicella-zoster virus

700
Q

Following primary infection with VZV (chicken pox) where does the virus lie dormant?

A

in the dorsal root or cranial nerve ganglia

701
Q

RFs for shingles?

A

age, HIV (15x more common), immunosupressions eg. steroids, chemo

702
Q

Most commonly affected dermatomes in shingles?

A

T1-L2

703
Q

Features of shingles?

A
  • prodromal period= burning pain over affected dermatome 2-3d; may be severe and interfere with sleep; 20% will have fever, headache, lethargy
  • Rash= initially erythematous, macular rash over affected dermatome; quickly becomes vesicular; well demarcated by dermatome and does not cross the midline, some ‘bleeding’ into adjacent areas may be seen
704
Q

Ix for shingles?

A

Clinical

705
Q

Mx for shingles?

A
  • advice
  • and analgesia: paracetamol & NSAIDs; if not helped then amitriptyline
  • antivirals within 72hrs unless pt <50yrs + mild truncal rash associated with mild pain and no underlying RFs e. aciclovir
706
Q

What Mx may be considered in first 2w in immunocompetent adults with localised shingles if pain severe and not responding to paracetamol, NSAIDs, or amitriptyline?

A

oral corticosteroids

707
Q

Advice for pt with shingles?

A

potentially infectious so…
- avoid pregnant women and immunosupressed
- infectious until vesicles have crusted over, usually 5-7d after onset
- covering lesions reduces risk

  • keep rash dry and clean
708
Q

Cx of shingles?

A
  • post-herpetic neuralgia (most common, older pt more, commonly resolves with 6m)
  • herpes zoster opthalmicus (affects ocular divsion of trigeminal nerve)
  • herpes zoster oticus (Ramsay Hunt syndrome)
709
Q

Should pt with shingles avoid school or work?

A

if rash is weeping and cannot be covered

710
Q

When to consider referral for shingles?

A

new vesicles forming after 7d antivirals, recurrent, pain not controlled or considering antivirals in pregnant/breastfeeding

711
Q

Shingles vaccination programme?

A

60yrs and older until their 80th bday
and >50yrs who are immunocompromised

sub-cut

live attenuated 1 dose eg. Zostavax
or
non-live 2 dose eg. Shingrix